Разное

Логические загадки с подвохом сложные: Загадки на логику для взрослых — сложные логические загадки с ответами

Содержание

Загадки на логику для взрослых — сложные логические загадки с ответами

В мифах Древней Греции есть история о храбром и мудром Эдипе, который правильно ответил на загадку страшного и непобедимого Сфинкса, тем самым сохранив свою жизнь. Острый ум и смекалистость всегда ценились в людях, вне зависимости от эпохи, в которой они родились и жили. Предлагаем раскрыть свой потенциал с помощью подборки интересных загадок, среди которых есть как смешные, так и сложные или с подвохом, поэтому будьте внимательны и не спешите сразу давать ответ.

Сложные логические загадки для взрослых

1. «Необычные цены»

Мужчина зашел в хозяйственный магазин и спросил цену на один товар. Продавец ему ответил:

  • один стоит 1 $;
  • восемь — 1 $;
  • тринадцать — 2 $;
  • сто шестьдесят восемь — 3 $;
  • четыре тысячи девятьсот восемьдесят шесть — 4 $.

Что покупал мужчина?

Показать ответ

Номерки на двери.

2. «Свет»

Вы стоите перед закрытой комнатой. Внутри помещения есть 1 лампочка накаливания. Окна и щели, чтобы разглядеть свет внутри комнаты, отсутствуют.

Справа от двери ведущей в комнату установлено 3 переключателя, которые находятся в положении «выкл», но только один из них подключен к лампе. Нужно узнать, какой переключатель включает лампочку, но при этом зайти в комнату можно только 1 раз.

Примечание: менять положение переключателей разрешено неограниченное количество раз.

Показать ответ

Включите сразу два любых переключателя, а один оставьте в положении «выкл». Подождите 2 минуты, выключите один из включенных переключателей и сразу войдите в комнату:

  • если лампа горит, значит, это тот переключатель, который остался включенным;
  • если света в комнате нет, потрогайте лампочку: горячая — рабочий переключатель, который был выключен перед входом в помещение, холодная — подключен переключатель, положение которого ни разу не менялось.

3. «Строитель и кирпичи»

У строителя есть 8 кирпичей, 7 из которых имеют одинаковый вес, а один — тяжелее. Как строителю выявить бракованный кирпич с помощью весов всего за два взвешивания?

Показать ответ

Все нужно делать поэтапно:

  1. Разделить 8 кирпичей на три стопки по 3, 3 и 2 штуки.
  2. Взвесить 2 стопки, в которых по 3 кирпича.
  3. Если они уравновесились, значит взвесить стопку из двух кирпичей и найти бракованный.
  4. Если не уравновесились, то взять 3 кирпича из стопки, которая перевесила и разделить на 3 штуки.
    Положить по одному на весы. Если один перевесил — он бракованный, если уравновесились, значит, тяжелее всех тот, что остался в стороне.

4. «Блюдо для Фараона»

Повару древнего Египта необходимо варить яйцо в кипящей воде строго две минуты, но у него в наличии песочные часы только на три, четыре и пять минут. Как ему приготовить блюдо Фараону, не отклоняясь от времени, которое указано в рецепте?

Примечание: использовать часы можно на свое усмотрение: сразу все, только одни, по очереди или по несколько раз.

Показать ответ

Когда вода закипит, повару нужно поставить двое часов: на три минуты и на пять. Когда песок в трехминутных часах закончится, сразу опускать яйцо, таким образом, оно будет вариться требуемое время.

5. «Женский возраст — это тайна»

Парень спросил у девушки: «Сколько тебе лет?», на что она загадочно улыбнулась и ответила: «Позавчера мне было 22, а в следующем году будет 25».

Попробуйте догадаться, когда у девушки День рождения и какого числа происходил разговор между парой?

Показать ответ

Парень и девушка разговаривали 1 января, а День рождения был 31 декабря. Получается:

  • «позавчера» было 30 декабря и девушке было 22 года;
  • 31-го числа ей исполнилось 23;
  • в День рождения, который будет в тот год, когда велся разговор, исполнится 24;
  • а на следующий год девушка отметит 25-летие.

6. «Загадочный треугольник»

Каким образом нарисовать треугольник, чтобы все его углы были 90 градусов?

Показать ответ

На мяче или любом другом сферическом предмете.

7. «Мудрый отец»

Сын попросил у отца 10 000 долларов на обучение, на что тот ответил, что не может ему дать их просто так, поскольку не уверен в его способностях и в том, что он сможет закончить экономический факультет. Но предложил пари.

Если сын разложит 1000 долларов по 10 конвертам таким образом, чтобы какую бы сумму отец не назвал, парень мог дать ему это количество денег одним или несколькими конвертами, но при этом не доставая и не докладывая деньги внутрь, то он получит 10 000 долларов.

Сын пришел через 30 минут с 10 запечатанными конвертами и отец назвал сумму 555 долларов, на что парень ему выдал три конверта. Открыв, отец пересчитал все деньги и в сумме оказалось ровно 555 долларов, после чего выписал чек на 10 000 долларов ему на обучение. Каким образом сын разложил деньги по 10 конвертам, чтобы можно было получить любую сумму до 1000 долларов включительно?

Показать ответ

Парень разложил в конверты такие суммы в долларах: 1, 2, 4, 8, 16, 32, 64, 128, 256, 489.

8. «Простое добавление»

Если одиннадцать плюс два равняются одному, чему равны девять плюс пять?

Показать ответ

Речь идет о времени: 11 часов плюс 2 часа = 1 час, а 9 часов плюс 5 часов = 2 часа.

9. «Восемь восьмерок»

Запишите восемь восьмерок таким образом, чтобы в сумме получилась тысяча.

Показать ответ

888 + 88 + 8 + 8 + 8 = 1000.

10. «Две нити»

У вас есть две одинаковые по длине нити, и известно, что при зажигании одного конца любой из них горение будет длиться ровно один час. Как с помощью всего двух таких ниток и спичек отмерить 45 минут, не прикасаясь при этом руками к ниткам?

Показать ответ

Зажгите оба конца первой нити и один конец второй. Когда полностью сгорит первая нить, это будет сигнализировать о прошествии 30 минут. Сразу же подпалите второй конец горящей нитки таким образом, чтобы огонь шел навстречу друг другу. В результате, когда она полностью сгорит, пройдет 45 минут.

Смешные загадки на логику

Как проходит разминка? Все получилось разгадать? Уверены, что да! А теперь попробуйте потренироваться на более веселых и смешных загадках, которые также могут стать отличным развлечением среди друзей большой компании.

1. «Несмелый ковбой»

Ковбой вбегает в бар и кричит: “Срочно мне налейте стакан воды!” Вместо этого бармен выхватывает пистолет и стреляет в воздух. Ковбой испугался, поблагодарил бармена и вышел. За что ковбой остался благодарен бармену?

Показать ответ

У ковбоя была икота, и когда бармен выстрелил, от испуга она прошла.

2. «Злостный нарушитель»

У женщины не было при себе водительского удостоверения. Она проехала без остановки на железнодорожном переезде при опущенном шлагбауме. Потом, проигнорировав «кирпич» двинулась во встречном направлении по улице с односторонним движением и остановилась только через три перекрестка. Сотрудники ГИБДД, видя все это, решили не вмешиваться. Почему?

Показать ответ

Женщина ехала на электричке, а потом шла пешком.

3. «Яркая профессия»

Олег зашел в магазин купить колбасы. Когда очередь дошла до него, он попросил порционно нарезать палку, при этом не поперек, а исключительно вдоль. Продавец поинтересовалась, не пожарником ли он случайно работает, на что Олег ответил: «Да». Как женщина догадалась о его профессии?

Показать ответ

Правильный ответ. Олег был в форме и вышел в обеденный перерыв.

4. «Практичные приметы»

Кому и при каких условиях не повезет встретить черную кошку «лицом к лицу»?

Показать ответ

Мыши. При любых условиях.

5. «Спортивный Санта»

Почему Санта Клаус так хорош в карате?

Показать ответ

Потому что у него черный пояс.

6. «Милые мишки»

Как называется медведь, у которого нет зубов?

Показать ответ

Мармеладный мишка.

7. «Слезы змеи»

Как заставить расплакаться гремучую змею?

Показать ответ

Забрать у нее погремушку.

8. «Конфуз в блошином цирке»

Почему собаку выгнали из блошиного цирка?

Показать ответ

Пес украл всех артистов шоу!

9. «Неожиданная поза»

Что происходит с курицей, когда она лежит вверх ногами?

Показать ответ

Ее приготовили и едят.

10. «Странная схожесть»

Что общего между ежиком и молоком?

Показать ответ

Они оба могут сворачиваться.

Логические загадки для взрослых с подвохом

А теперь попробуйте мыслить нестандартно и креативно, стараясь заметить мелочи и не повестись на уловки, ведь далее представлена подборка загадок с подвохом.

1. «Непонятный возраст»

Одной женщине в 1850 году исполнилось 40 лет, но в 1860 году ей было 30 лет. Как такое могло получиться?

Показать ответ

В 1850 году женщине только исполнилось 40 лет, в то же время в 1860 году ей уже было 30, получается 1860 год был раньше чем 1850, а такое возможно только в том случае, если летоисчисление ведется до рождества Христова, то есть 1850 год до н. э. и 1860 год до н. э.

2. «Игра до рассвета»

Четверо мужчин сели играть. Игра длилась всю ночь до самого рассвета. Причем все они играли за деньги, а не ради удовольствия. Когда пришло время сводить счета, мужчины заработали приличные суммы и ни один из них не проиграл.

Как такое могло выйти?

Показать ответ

Эти мужчины — члены оркестра, нанятого для частной вечеринки.

3. «Правила дорожного движения»

Перед перекрестком на светофоре стоят и ждут зеленого света: фура, телега с лошадью и мотоциклист. Загорелся желтый, водитель фуры нажал на газ, отчего лошадь испугалась и укусила за ухо мотоциклиста. Кто из троих нарушил ПДД?

Показать ответ

Мотоциклист, так как был без шлема.

4. «Человек дождя»

Художник гулял по парку, когда на улице начался дождь. С собой у мужчины не оказалось шляпы и зонтика, а в кронах деревьев из-за сильного ливня укрыться не вышло. В результате вся одежда оказалась влажной, но ни один волос на голове художника не промок. При каких обстоятельствах такое могло произойти?

Показать ответ

Это был лысый художник.

5. «Провинившийся провидец»

Находясь в лагере, развернутом на поле во время военных действий, солдат был в ночном карауле. Около 4-х часов утра он разбудил командующего и сообщил, что этой ночью увидел во сне, как противники атакуют их лагерь с юга. И хотя командующий не был суеверным человеком, все же выставил наблюдателей за южной частью равнины.

Спустя некоторое время, противник напал именно с указанной солдатом стороны, но так как меры были предприняты заранее, войско смогло с легкостью отбить атаку и напасть в ответ на противника, победив его. После изнурительного боя и одержанной победы командующий поблагодарил солдата и посадил его в тюрьму. Почему?

Показать ответ

Сны человеку снятся, только когда он спит, а караульному это запрещено.

6. «Смекалистый отличник»

Перед началом экзамена профессор объявил студентам: «На этом письменном экзамене есть три вопроса. У вас будет всего один час, чтобы ответить на все, и не более. Любой, кто будет писать дольше, получит двойку без права на пересдачу!».

Андрей, получив свой билет, принялся отвечать. После того как пропищал таймер, осталось всего 5 минут, студент понял, что ответил только на два вопроса и часть третьего. Осмотревшись в аудитории, парень заметил, что с их группой сдает другой экзамен старший курс, а значит, ему может сойти с рук несколько лишних минут, пока другие будут сдавать работы.

Закончив отвечать на вопрос, Андрей пошел сдавать работу, но по дороге его перехватил профессор и сказал: «Я видел, как после завершения одного часа, ты продолжал нагло писать, а потому заслуживаешь только двойку!»

Парень подумал и спросил: «Вы знаете, кто я?» На что профессор покачал головой, подтвердив, что даже не имеет представления, кто он такой, и ответил: «Меня не волнует, кто ты такой. А вот тебе нужно уважать меня и соблюдать дисциплину!»

– Хорошо! — ответил Андрей, положил работу и ушел. В день объявления результатов он получил оценку «отлично». Почему?

Показать ответ

Когда профессор подтвердил, что не знает, кто такой Андрей, тот быстро сунул экзаменационную работу в общую стопку на столе и ушел, чтобы профессор не знал, какая работа именно его и заслуживает неудовлетворительной оценки за несвоевременную сдачу. Поэтому во время проверки это требование профессор просто не учел и оценивал лишь знания студента, которые показали отличный результат!

7. «Знаменитости»

Наполеон Бонапарт, Петр Первый, Юрий Гагарин, Шерлок Холмс, Вильям Шекспир, Нострадамус, Пифагор. Кто лишний в этом списке?

Показать ответ

Шерлок Холмс, поскольку это вымышленный персонаж, а остальные — реальные люди.

8. «Восемь подружек»

Восемь подружек находятся в доме и каждая из них занята каким-то делом:

  • первая — готовит ужин;
  • вторая — разжигает камин;
  • третья — читает книгу;
  • четвертая — играет в шахматы;
  • пятая — рисует картину;
  • шестая — убирается в доме;
  • седьмая — вешает шторы.

Чем занята восьмая девочка?

Показать ответ

Играет в шахматы вместе с четвертой подружкой.

9. «Прогноз погоды»

Если в 12 часов ночи пошел снег, можно ли предположить, что через 96 часов на улице будет солнце?

Показать ответ

Такого быть не может так как 96 часов — это ровно четверо суток, а значит, через указанный промежуток времени снова будет 12 часов ночи.

10. «Четыре сестры»

У мамы Мэри четыре дочери. Имя первой — Априна, второй — Мая, третьей — Июния.

Как зовут четвертую девочку?

Показать ответ

Ее зовут Мэри.

Математика и логика для детей 7-13 лет

Развиваем логическое мышление через решение сюжетных математических задач в интерактивном игровом формате

узнать подробнее

Самые крутые загадки. Логические и занимательные задачи (300 задач)

Приветствую друзья! Сегодня я предлагаю окунуться в мир веселья и загадок. Моя огромная подборка пойдет для детей и взрослых, в ней собраны сложные загадки на логику, короткие загадки с подвохом и, разумеется, они все с ответами.

Чтобы развивать логику, не надо выдумывать новые детективные загадки, не стоит сочинять криминальные, достаточно воспользоваться тем, что вы нашли или тем, что у вас есть в наличии. Еще наши родители любили за застольями и во время веселых встреч загадывать советские загадки, только тогда с подвохом не сильно пользовались популярностью.

Друзья, в этот раз у меня собралось воедино много русских и смешных загадок, так же серьезных и сложных, которые подойдут для детей старше 12 лет и взрослых. Это не первая моя подборка, остальные вы сможете найти в рубрике «творчество поделки». Надеюсь вы оцените мои старания. Для более легкого ориентирования, я разбила интересные загадки на подгруппы.

Сырым не едят, вареным выбрасывают. Что это?

(лавровый лист).

***
Без чего ничего никогда не бывает?

(без названия).

***
Сидит девушка, а вы не можете сесть на ее место, даже если она встанет и уйдет. Где она сидит?

(Она сидит у вас на коленях).

***
Как человеку не спать 8 дней?

(спать по ночам).

***
Стоит богатый дом и бедный. Они горят. Какой дом будет тушить полиция?

(Полиция пожары не тушит, пожарные тушат пожары).

***
Чтобы найти зарытый пиратский клад, нужно пройти от старого дуба 12 шагов на север и 5 шагов на юг, затем 4 на север и 11 на юг. Где зарыт клад? (у старого дуба).

***
— Она красная?

— Нет, черная.

— А почему она белая?

— Потому, что зеленая.

(черная смородина).

***
Когда мы смотрим на цифру 22, а говорим 10?

(когда смотрим на часы).

Загадки на логику с ответами не смешные, но очень хорошо воздействуют на мозг ребенка и взрослого. Некоторые из предоставленных, можно отнести к категории для взрослых, ведь не каждому школьнику под силу отгадать подобное. В продолжении я добавлю еще несколько математических загадок на логику с ответами.

На столе лежит 1 апельсин. Его разрезали на 2 части, сколько апельсинов лежит на столе?

(1 разрезанный).

***
На опушке стояло 3 высоких сосны. На каждой сосне по 3 больших ветки и по 3 маленьких. На каждой маленькой ветке по яблоку. Сколько всего яблок на деревьях?

(0, на соснах яблоки не растут).

***
В поле работали 5 тракторов. 2 трактора сломались и остановились. Сколько тракторов в поле? (5, так как учитываются все тракторы, и рабочие, и сломанные).

***
Марина шла из дома в школу и встретила трех мужиков. У каждого за спиной был мешок. У первого мужика в мешке был один кот, у второго в мешке был один кот и один пес. У третьего в мешке было 2 пса. Сколько всего людей и животных направлялось в школу?

(Один человек, сама Марина. Мужики с мешками шли в обратную от школы сторону).

Загадки на логику с ответами, с подвохом

Смешные загадки на логику с ответами любят дети любого возраста и в 7 лет и в 10, при помощи моей подборки вы отлично проведете время в кругу семьи. Поиграйте в подобные логические и интересные задачи с родителями, взрослым это тоже интересно.

На обед сыночку Ване

Мама варит суп в

(не в стакане, а в кастрюле)

***
На дорогах стало суше – У меня сухие

(не уши, а ноги)

***
И капризна, и упряма,

В детский сад не хочет

(не мама, а дочка)

***
Ни к чему весь этот спор – резать ткань бери

(не топор, а ножницы)

***
Хоккеистов слышен плач

Пропустил вратарь их

(не мяч, а шайбу)

***
Ночью каждое оконце

Слабо освещает

(Луна, не солнце).

***
Я скажу вам без затей —

В сумке я ношу детей.

А в прыжках я чемпион,

Я прыгучий серый

(Кенгуру, а не слон).

***
От рыжей лисицы

В кустарник удрав

В листву завернулся

Колючий

(еж, а не удав).

***
Просит бабушка Аркашу

Из редиски скушать

(Салат, а не кашку).

***
Попросила мама Юлю

Ей чайку налить в

(Чашку, а не кастрюлю).

***
Синим лаком захотела

Я себе покрасить

(Ногти, а не тело).

***
Черный весь, как будто грач,

С нашей крыши лезет

(Трубочист, а не врач).

***
С каждым днем к нам лето ближе,

Скоро встанем все на

(Ролики, а не лыжи).

***
На уроках будешь спать – за ответ получишь

(Два, а не 5).

Сложные загадки на логику с ответами

Два пятиклассника Петя и Алёнка идут со школы и разговаривают.

Когда послезавтра станет вчера, – сказал один из них, – то сегодня будет так же далеко от воскресенья, как и тот день, который был сегодня, когда позавчера было завтра. В какой день недели они разговаривали? (воскресенье).

***
Кошка – 3, лошадь – 5, петух – 8, ослик – 2, кукушка – 4, лягушка – 3, собака -? О чем идет речь?

(Кошка — мяу(3), Лошадь — и-го-го(5), Петух — ку-ка-ре-ку(8), Ослик — и-а(2),…, Собака — гав(3) – буквы).

***
На постоялый двор приехал путешественник. Денег у него с собой не было, но была серебряная цепочка, состоящая из шести звеньев. Хозяин отеля согласился принять в оплату номера за каждый день по одному колечку с этой цепочки, но так, чтобы распиленных колец он получил не более одного. Как нужно путешественнику распилить цепочку, чтобы можно было расплачиваться с хозяином за каждый день отдельно в течение пяти дней?

(Достаточно распилить одно третье звено. За первый день он расплатится распиленным звеном. За другой день – отдаст два звена, получив в качестве сдачи распиленное звено. За третий день пребывания заплатит куском из трех звеньев, получит сдачу – два звена).

***
Недалеко от берега стоит корабль со спущенной на воду веревочной лестницей, которая имеет 10 ступенек. Расстояние между ними составляет 30 см. Самая нижняя ступенька касается поверхности воды. Океан пока спокойный, но чувствуется начало прилива, который поднимет воду за час на 15 см. Через, сколько часов покроется водой третья ступенька веревочной лестницы?

(Вода никогда не покроет третьей ступеньки. Вместе с водой поднимется и лестница).

***
Что человеку дается бесплатно дважды, а если он захочет иметь это в третий раз, то приходится платить?

(Зубы).

***
В кастрюле налита вода до самого верха. Как отмерять жидкость, не используя никаких мерительных приспособлений, чтобы в кастрюле осталась только половина жидкости.

(Нужно наклонить кастрюлю и выливать воду до тех пор, пока не покажется с боковой части дно. Это и будет половина кастрюли).

***
Здорового человека, который не болеет, не имеет инвалидности и у которого все в порядке с ногами, выносят на руках из больницы. Кто это? (Новорожденный ребенок)
***
Ты на соревновании по бегу. Когда ты перегнал того, кто бежал последним, каким ты стал?

(Такого не может быть, ведь последнего бегуна нельзя перегнать, потому что он последний и сзади него не может быть еще кого-то).

Не скажу, что это самые сложные загадки на логику с ответами, но чтобы их разгадать без подсказки, придется немного потрудиться. Эта категория рассчитана для взрослых и если вы справились с задачей на логику, поделитесь этим с другими.

Короткие загадки на логику с ответами

Простой вопрос для малышей:

Кого боится кот?

(Собак, а не мышей).

***
Два брюшка – четыре ушка

(Подушка).

***
К реке подходят два человека. У берега лодка, которая может выдержать только одного. Оба человека переправились на противоположный берег. Как?

(Они были на разных берегах).

***
Как может брошенное яйцо пролететь три метра и не разбиться?

(Нужно бросить яйцо на четыре метра, тогда первые три метра оно пролетит целым).

***
Если пять кошек ловят пять мышей за пять минут, то сколько времени нужно одной кошке, чтобы поймать одну мышку?

(Пять минут).

***
Как разделить пять яблок между пятью девочками так, чтобы каждая получила по яблоку, и при этом одно из яблок осталось в корзинке?

(Отдать одной девочке яблоко вместе с корзинкой).

***
У бабушки Даши внук Паша, кот Пушок, собака Дружок. Сколько у бабушки внуков?

(Один).

***
Что легче 1 кг ваты или 1 кг железа?

(Одинаково).

***
Что с пола за хвост не поднимешь?

(Клубок ниток).

Загадки в картинках на логику с ответами


Загадки на логику с ответами 7 лет

Что не может поместиться даже в самую большую кастрюлю?

(Ее крышка).

***
Под крышей – четыре ножки, а на крыше – суп да ложки

(Стол).

***
В нашем доме под окошком есть горячая гармошка: не поет и не играет – она дом обогревает (Батарея отопления).

***
Когда шесть детей, трое взрослых и две кошки не промокнут все вместе под зонтом?

(Если не будет дождя).

***
Один шофёр не взял с собой водительские права. Был знак одностороннего движения, но он двинулся в обратном направлении. Полицейский это видел, но не остановил его. Почему?

(Шофёр шёл пешком).

***
Каким гребешком нельзя расчесаться?

(Петушиным).

***
Что станет с красным мячиком, если он упадёт в Чёрное море?

(Он намокнет).

***
Какой нос не чувствует запаха?

(Нос ботинка или сапога, носик чайника).

***
Это ленточка такая – шелковисто-расписная,

Но руками не возьмешь и в косички не вплетешь.

(Радуга).

***
Где король и королева ходят вправо, ходят влево,

Ходят – прямо, ходят – косо, ходят – молча, без вопросов.

Даже целый день подряд по желанию ребят?

(На шахматной доске).

***
Любят, с искренностью всей, в дом пускать они гостей.

Но в гостях, скажу вам сразу, сами не были ни разу.

(Двери).

Интересные загадки для всей семьи, это весело и отличное времяпровождение. Развивайте логику и мышление при помощи загадок.

Загадки на логику 10 лет с ответами

Человек спускался по лестнице, которая насчитывала семь ступенек, и сломал ногу. Если бы лестница имела не семь, а двадцать восемь ступеней, сколько ног он сломал бы?

(Ни одной, или еще одну)

***
Что общего между числами 69 и 88?

(Если их перевернуть они будут выглядеть также).

***
На край стола поставили кастрюлю, плотно закрытую крышкой, таким образом, что две трети кастрюли свисало со стола. Через некоторое время кастрюля упала. Что в ней было?

(Лёд).

***
В 9-этажном доме есть лифт. На первом этаже живут 2 человека, на втором — 4 человека, на третьем — 8 человек, на четвёртом — 16, на пятом — 32 и т. д. Какая кнопка в лифте этого дома нажимается чаще других?

(Кнопка первого этажа).

***
Шел солдат мимо Эйфелевой башни. Достал ружьё и выстрелил. Куда он попал?

(В полицию).

***
Когда строят дом, гвоздь во что вбивают?

(В шляпку).

***
Что идёт то в гору, то с горы, но остаётся на месте?

(Дорога).

Загадки на логику с ответами 12 лет

Папу Антона зовут Андрей Викторович, а дедушку – Сергей Иванович. Какое отчество у мамы Антона?

(Сергеевна, Потому что Сергей Иванович – это отец мамы Антона. Отца папы Антона зовут Виктор).

***
Что бросают, когда нуждаются в этом, и поднимают, когда в этом нет нужды?

(Морской якорь).

***
Шли два отца и два сына, нашли три апельсина. Стали делить – всем по одному досталось. Как это могло быть?

(Это были дед, отец и сын).
***
В комнате было 12 цыплят, 3 кролика, 5 щенят, 2 кошки, 1 петух и 2 курицы. Сюда зашёл хозяин с собакой. Сколько в комнате стало ног?
(Две, у животных лапы).

***
Укажите пять дней, при этом, не обозначая их числами и не называя дня недели.

(Сегодня, вчера, позавчера, завтра, послезавтра).

***
Что одновременно делают все люди на Земле?

(Живут).

***
Оно перед нами всегда, но увидеть его мы не можем.

(Будущее).

***
Какой рукой лучше размешивать чай?

(Той, в которой ложка).

***
Как долго можно идти в лес?

Друзья, ну как вам мои загадки для детей на логику, возможно, они не самые сложные, но поразмышлять все – таки придется. Советские загадки знают многие, а тут подобраны современные для детей и взрослых. Делитесь ими в социальных сетях, буду вам искренне благодарна.

Ваша Нина Кузьменко!

АРИФМЕТИЧЕСКИЕ И ЛОГИЧЕСКИЕ ЗАГАДКИ

У бабушки Даши внук Паша, кот Пушок, собака Дружок. Сколько у бабушки внуков?

Термометр показывает плюс 15 градусов. Сколько градусов покажут два таких термометра?

Саша тратит на дорогу в школу 10 минут. Сколько времени он потратит, если пойдёт вместе с другом?

Ребёнок моего отца, мне не брат. Кто это?

В парке 8 скамеек. Три покрасили. Сколько скамеек стало в парке?

Меня зовут Юра. У моей сестры только один брат. Как зовут брата моей сестры?

Батон разрезали на три части. Сколько сделали разрезов?

Что легче 1 кг ваты или 1 кг железа?

(Одинаково)

Грузовик ехал в деревню. По дороге он встретил 4 легковые машины. Сколько машин ехало в деревню?

Два мальчика играли в шашки 2 часа. Сколько времени играл каждый мальчика

(Два часа)

Мельник пошёл на мельницу и увидел в каждом углу по 3 кошки. Сколько ног на мельнице?

Знаменитый фокусник говорит, что может поставить бутылку в центре комнаты и вползти в неё. Как это?

(Вползти в комнату может каждый)

Один шофёр не взял с собой водительские права. Был знак одностороннего движения, но он двинулся в обратном направлении. Полицейский это видел, но не остановил его. Почему?

(Шофёр шёл пешком)

Может ли дождь лить два дня подряд?

(Нет, между ними – ночь)

Что будет с вороной, когда ей исполнится 7 лет?

(Пойдёт восьмой)

Запрыгнуть на ходу в него можно, а выпрыгнуть на ходу из него нельзя. Что это?

(Самолёт)

Два раза родится, один раз умирает. Кто это?

(Цыплёнок)

Что с пола за хвост не поднимешь?

(Клубок ниток)

Кто ходит сидя?

(Шахматист)

Что всегда увеличивается и никогда не уменьшается?

(Возраст)

На край стола поставили кастрюлю, плотно закрытую крышкой, таким образом, что две трети кастрюли свисало со стола. Через некоторое время кастрюля упала. Что в ней было?

Чем больше из неё берёшь, тем больше становится… Что это?

Девочка упала со второго этажа и сломала ногу. Сколько ног сломает девочка, если упадёт с четвёртого этажа?

(Максимум одну, так как вторая нога уже сломана)

Мальчик идёт из школы домой 30 минут. За сколько минут эту же дорогу пройдут 3 мальчика?

(За 30 минут)

Где оказался Моисей, когда погасла свеча?

(В темноте)

В 9-этажном доме есть лифт. На первом этаже живут 2 человека, на втором – 4 человека, на третьем – 8 человек, на четвёртом – 16, на пятом – 32 и т. д. Какая кнопка в лифте этого дома нажимается чаще других?

(Кнопка первого этажа)

Когда чёрной кошке лучше всего пробраться в дом?

(Когда дверь открыта)

Шел солдат мимо Эйфелевой башни. Достал ружьё и выстрелил. Куда он попал?

(В полицию)

Когда строят дом, первый гвоздь во что вбивают?

(В шляпку)

Что идёт то в гору, то с горы, но остаётся на месте?

На что больше всего похожа половина апельсина?

(На вторую половину апельсина)

Двое пошли – три груздя нашли. Следом четверо идут, много ли груздей найдут?

(Ни одного)

В коробке 25 кокосовых орехов. Обезьяна стащила все орехи, кроме 17. Сколько орехов осталось в коробке?

(Осталось 17 орехов)

К тебе пришли гости, а в холодильнике – бутылка лимонада, пакет с ананасовым соком и бутылка минеральной воды. Что ты откроешь в первую очередь?

(Холодильник)

Каким гребешком нельзя расчесаться?

(Петушиным)

В каком месяце 28 дней?

(В каждом месяце ест 28-е число)

Что сырым не едят, а сварят – выбросят?

(Лавровый лист)

Какой месяц короче всех?

(Май – в нём всего три буквы)

Что станет с красным мячиком, если он упадёт в Чёрное море?

(Он намокнет)

Какой рукой лучше размешивать чай?

(Лучше все-таки размешивать ложечкой)

На какой вопрос нельзя ответить “да”?

(“Ты спишь?”)

На какой вопрос нельзя ответить “нет”?

(“Ты жив?”)

Какой нос не чувствует запаха?

(Нос ботинка или сапога, носик чайника)

Сколько яиц можно съесть натощак?

(Одно. Все остальное будет съедено уже не натощак)

Тебе дано, а люди пользуются. Что это?

Может ли страус назвать себя птицей?

(Нет, так как он не умеет говорить)

Мужчина ехал в машине. Фары он не включил, луны тоже не было, фонари вдоль дороги не светили. Перед машиной дорогу стала переходить старушка, но водитель вовремя затормозил, и аварии не произошло. Как ему удалось разглядеть старушку?

(Был день)

Какое ухо не слышит?

(Ухо (ушко) у кружки)

Что можно увидеть с закрытыми глазами?

Как долго можно идти в лес?

Сын моего отца, а мне не брат. Кто это? Где вода столбом стоит? Что видно только ночью?

(Я сам) (В колодце) (Звёзды)

Летела стая уток: две впереди, две позади, одна посередине и три в ряд. Сколько их всего?

Шли столбцом сын с отцом да дед с внуком. Сколько их?

Шёл муж с женой, брат с сестрой да шурин с зятем. Много ли всех?

Летела стая птиц, сели по двое на дерево – одно дерево осталось; сели по одной – одного недостало. Сколько птиц и сколько деревьев?

(Три дерева и четыре птицы)

На что купец шляпу купил?

(На деньги)

По какой дороге полгода ездят и полгода ходят?

Кто из нас не любит загадки? Дети загадывают загадки друг другу, чтобы удивить своих сверстников и повеселиться. Загадки с подвохом нравятся и взрослым. Если загадка интересная, оригинальная, каверзная, ну, как не повеселить себя и друзей. Ведь загадка – это народное творчество, в котором отражается наша реальность и способ ее восприятия. Именно на то, как мы воспринимаем окружающую нас бытовую жизнь, и опираются загадки с подвохом. Логика нашего восприятия такова, что мы часто используем в мышлении уже готовые шаблоны, заданные жизнью логические цепочки. И когда загадка указывает нам на это своим неожиданным ответом, это нас порядком удивляет и веселит.

Не мудрено, что во многих народах загадка – широко распространенный фольклорный жанр. Вспомните, как часто они встречаются в мифах и сказках. Герой повествования должен отгадать ряд хитрых загадок, чтобы получить желаемое. Такие сказки высмеивают отсутствие смекалки, учат смотреть на жизнь с разного ракурса, усматривать сказочное в обыденном. Обычно загадки делаются изначально с подвохом, чтобы ответ лежал на поверхности, но не был очевиден.

На этой странице вы найдете множество смешных загадок с подвохом. Ну и, конечно же, ответы на них. Ведь интересная загадка без ответа может свести с ума.

Сложные загадки с подвохом на логику

Загадки на логику с подвохом, на абстрактное мышление помогают разрушить старые представления об обычных вещах, лучше узнать себя и взглянуть на знакомые ситуации под иным углом. Ответ на загадку с подвохом всегда парадоксален, в этом ее суть.
Тем не менее, если вы посмотрите на приведенные ниже загадки, то увидите, что не все из них можно разгадать, используя логический подход. Иногда требуется просто хорошее чувство юмора.

Есть несколько основных принципов, по которым складываются загадки с подвохом. Для того, чтобы запутать отгадчика в тексте загадки применяются преувеличения, отрицания, сравнения живого с неживым. Именно сравнения и метафоры – суть загадок с подвохом. Метафора сравнивает несравнимое, при этом скрывая от восприятия важные для отгадки факторы.

А почему загадки с подвохом обычно нас смешат? Да потому что на мудреный, казалось, вопрос находится простой и необычный ответ. Когда наша мысль идет по заданному руслу, обдумывает различные серьезные комбинации, ответ оказывается на поверхности. И эта неожиданная простота вызывает смех.

Трудные загадки с подвохом:

Загадки полезны детям и взрослым. Они развивают сообразительность, абстрактное мышление и внимание. Они учат нас пониманию того, что одна и та же мысль может быть сформулирована различными способами. Загадки с подвохом тренируют память и речевой слух. А самое главное, они дарят хорошее настроение.

Дорогие наши читатели! Чтобы начало рабочей недели не казалось таким тоскливым и чтобы вы могли настроиться на рабочую волну, почувствовать себя в тонусе, блог За городом собрал для вас 15 интересных загадок с подвохом, отгадать которые с первого раза может и не получится. Отгадки мы написали внизу. Только, чур, не подсматривать. Иначе будет не интересно. Просыпаемся и находим ответы самостоятельно))

1. К реке подходят два человека. У берега лодка, которая может выдержать только одного. Оба человека переправились на противоположный берег. Как?

2. Где встречается такое, что конь через коня перепрыгивает?

3. Шерлок Холмс шел по улице.И вдруг он увидел мертвую женщину лежащую на земле. Он подошел, открыл ее сумку и достал телефон. В тел. книге он нашел номер ее мужа. Он позвонил. Говорит:
– Срочно приезжайте сюда. Ваша жена умерла. И через некоторое время муж приезжает. Он смотрит на жену и говорит:
– О, милая что с тобой случилось???
И потом приезжает полиция. Шерлок показывает пальцем на мужа женщины и говорит:
– Арестуйте этого человека. Это он убил ее. Вопрос: Почему Шерлок так подумал?

4. Банка стоит на столе. Стоит она так, что одна её половина находится в воздухе а другая на столе. Что лежит в банке, если через полчаса она упадёт? И почему?

5. Отправился человек в море и попал в шторм. Его отнесло на остров, где не было мужчин, а жили только девушки. Утром проснулся он весь в веревках на каком-то ритуале и узнал, что его хотят убить. И попросил он последнее слово. После того, как он сказал его, девушки смастерили ему лодку, дали еду, воду и отправили домой. Что он сказал?

6. Эту загадку ученик 1-ого класса решает за 5 минут, старшеклассник за 15 минут, студент за 1 час, профессор никогда не решит. Загадка: расшифруйте одтчпшсвдд

7. Один поезд едет из Москвы в С.-Петербург с опозданием 10 минут, а другой – из С.-Петербурга в Москву с опозданием 20 минут. Какой из этих поездов будет ближе к Москве, когда они встретятся?

8. Известно, что среди девяти монет есть одна фальшивая, у которой вес меньше чему у остальных. Как с помощью чашечных весов за два взвешивания определить фальшивую монету?

9. Есть два шнура, каждый из которых горит по часу, но горит неравномерно. Как с помощью этих двух шнуров и спичек отмерить 45 минут?

10. На гладкую доску положили 2 кирпича – один плашмя, а другой на ребро. Кирпичи весят одинаково. Какой кирпич соскользнет первым, если наклонять доску?

11. Кошка – 3 Лошадь – 5 Петух – 8 Ослик – 2 Кукушка – 4 Лягушка – 3 Собака -?

12. Повстречались три преступника: медвежатник Белов, домушник Чернов и карманник Рыжов. “Удивительно то, что один из нас имеет черные, второй белые, а третий рыжие волосы, но ни у одного цвет волос не совпадает с фамилией”, – сказал черноволосый. “И правда…”, – сказал медвежатник Белов. Какой у карманника?

13. Вы стоите перед тремя выключателями. За непрозрачной стеной три лампочки в выключенном состоянии. Вам нужно произвести манипуляции с выключателями, зайти в комнату и определить, к какой лампочке относится какой выключатель.

14. Стоит стена из бетона высотой в 3 метра, длиной в 20 метров и весом в 3 тонны. Как ее повалить, не имея никаких вспомогательных средств и инструментов?

15. Отец с двумя сыновьями отправился в поход. На их пути встретилась река, у берега которой находился плот. Он выдерживает на воде или отца, или двух сыновей. Как переправиться на другой берег отцу и сыновьям?

Ответы:

1. Они были на разных берегах.
2. В шахматах.
3. Потому что Холмс не сказал ему адрес.
4. Лёд
5. Пусть меня убьет самая некрасивая.
6. 1,2,3,4…
7. В момент встречи они будут на одинаковом расстоянии от Москвы.
8. 1-е взвешивание: 3 и 3 монеты. Фальшивая монета в той кучке, которая меньше весит. Если равны, то фальшивка в третьей кучке. 2-е взвешивание: Из кучки с наименьшим весом сравниваются 1 и 1 монета. Если равны, то фальшивка – оставшаяся монета.
9. Надо поджечь первый шнур одновременно с обоих концов – это 30 минут. Одновременно с первым шнуром поджигаем второй шнур с одного конца, и когда первый шнур догорит за 30 минут, поджигаем второй шнур с другого конца – получаем оставшиеся 15 минут.
10. Кирпичи начнут скользить одновременно. Оба кирпича давят на доску с одинаковой силой, а значит, одинаковы и силы трения, которые приходится им преодолевать. Удельные силы трения, приходящиеся на каждый квадратный сантиметр площади соприкосновения кирпичей с доской, естественно, не равны. Но общие силы трения, действующие на кирпичи, равные произведению удельной силы трения на площадь поверхности соприкосновения, будут одинаковы.
11. Кошка – мяу(3), Лошадь – и-го-го(5), Петух – ку-ка-ре-ку(8), Ослик – и-а(2),…, Собака – гав(3)
12. Белов – не белый из-за фамилии и не черный, так как он ответил черноволосому. То есть Белов – рыжий. Чернов не черный из-за фамилии и не рыжий, так как рыжий у нас медвежатник Белов. Карманнику Рыжова остался черный цвет.
13. Включить два выключателя. Через какое-то время один выключить. Зайти в комнату. Одна лампочка будет горящая от включенного выключателя, вторая горячая – от включенного и выключенного, третья – холодная, от нетронутого
14. Толщина такой стены будет не более двух сантиметров, что позволяет толкнуть ее рукой
15. Вначале переправляются оба сына. Один из сыновей возвращается обратно к отцу. Отец перебирается на противоположный берег к сыну. Отец остается на берегу, а сын переправляется на исходный берег за братом, после чего они оба переправляются к отцу.

Чем лошадь отличается от иголки?

(На иголку сначала сядешь, потом подпрыгнешь, а на лошадь сначала подпрыгнешь, потом сядешь.)

Черная собачка не лает,
не кусает, а в дом не пускает.

(Дохлая черная собака, загородившая вход в дом)

Домашнее животное, на “т” начинается.

(Таракан)

Домашнее животное, на “д” начинается.

(Два таракана)

Домашнее животное, на “ы” начинается.

(Ыщо один таракан)

(Нет, – через 72 часа будет снова полночь)

Сколько горошин может войти в один стакан?

(Нисколько, т.к. горошины не ходят)

Бежит ежик по лужайке – тащится, хохочет. Почему хохочет?

(Потому что травка писю щекочет)

Бежит ежик по лужайке – плачет. Почему плачет?

(Травку скосили)

Два гвоздя упали в воду. Как фамилия грузина?

(Заржавели)

Летел по небу бегемотик, а по земле за ним бежал охотник с ружьем. Охотник
выстрелил, и бегемотик упал на него. Кто жив остался?

(Слоник, потому что он вылетел позже)

На край стола поставили жестяную банку, плотно закрытую крышкой, так, что 2/3 банки свисало со стола. Через некоторое время банка упала. Что было в банке?

(Кусок льда)

(Да, гальванический)

Как известно, все исконно русские женские имена оканчиваются либо на “а”, либо на “я”: Анна, Мария, Ольга и т.д. Однако есть одно-единственное женское имя, которое не оканчивается ни на “а”, ни на “я”. Назовите его.

Назовите пять дней, не называя чисел (напр., 1, 2, 3,..) и названий дней (напр., понедельник, вторник, среда…)

(Надо достать лист бумаги)

Один поезд едет из Москвы в С.-Петербург с опозданием 10 минут, а другой – из С.-Петербурга в Москву с опозданием 20 минут. Какой из этих поездов будет ближе к Москве, когда они встретятся?

(Эта задача в компании сразу выявляет физика: физик сразу отвечает, что ей нужно бежать со сверхзвуковой скоростью. Разумеется, собаке достаточно стоять на месте)

(1 ч 40 мин = 100 мин)

Крыша одного дома не симметрична: один скат ее составляет с горизонталью угол 60 градусов, другой – угол 70 градусов. Предположим, что петух откладывает яйцо на гребень крыши. В какую сторону упадет яйцо – в сторону более пологого или крутого ската?

(Петухи не кладут яйца)

В 12-этажном доме есть лифт. На первом этаже живет всего 2 человека, от этажа к этажу количество жильцов увеличивается вдвое. Какая кнопка в лифте этого дома нажимается чаще других?

(Всего одну, т.к. вторая у него уже сломана, либо не больше ни одной, если повезёт;-)

Шел Кондрат в Ленинград,
А навстречу – двенадцать ребят,
У каждого по три лукошка,
В каждом лукошке – кошка,
У каждой кошки – двенадцать котят,
У каждого котенка в зубах по четыре мышонка.
И задумался старый Кондрат:
“Сколько мышат и котят
Ребята несут в Ленинград?”

                        (Глупый, глупый Кондрат!
                        Он один и шагал в Ленинград.
                        А ребята с лукошками,
                        С мышками и кошками
                        Шли навстречу ему – в Кострому)

(Да, это всадник на лошади)

(Запасное)

Еще одна загадка “с бородой”: шли два отца и два сына, нашли три апельсина. Стали делить – всем по одному досталось. Как это могло быть?

(Это были дед, отец и сын)

(На березе яблоки не растут)

(Под мокрым)

(Слово “неверно”)

(Из пустой)

(На другую сторону дороги)

Слово из трех букв, которого боится любой мужчина?

Что такое: самое доброе в мире приведение с моторчиком?

(Запорожец)

А любит Б, Б любит Ц?
Что делать А?

(Найти другую Б)

Что можно приготовить, но нельзя съесть?

(Да много чего: домашнее задание, цемент)

Как можно поместить два литра молока в литровую бутылку?

(Налить в бутылку литр, когда его выпьют, налить второй литр; или насыпать сухого молока…)

Если пять кошек ловят пять мышей за пять минут, то сколько времени нужно одной кошке, чтобы поймать одну мышку?

Сколько месяцев в году имеют 28 дней?

(Все 12, т.к. если в месяце 30 дней, то и 28 дней среди них есть)

Что бросают, когда нуждаются в этом, и поднимают, когда в этом нет нужды?

(Якорь (морской, а не ресурсный;)

Собака была привязана к десятиметровой веревке, а прошла триста метров. Как ей это удалось?

(Она ходила внутри круга с радиусом 10м, причём не обязательно по кругу)

Что может путешествовать по свету, оставаясь в одном и том же углу?

(Палец на карте, глобусе; марка на конверте; интернетчик;-)

Можно ли зажечь спичку под водой?

(Если ты в подводной лодке, то да.

Как может брошенное яйцо пролететь три метра и не разбиться?

(Главное бросать его так, чтобы оно летело больше 3 метров, тогда оно разобьется не когда пролетит 3м, а когда упадет)

Что станет с зеленым утесом, если он упадет в Красное море?

(Ничего, разве что немного раскрошится от падения, или утонет)

Мужчина вел большой грузовик. Огни на машине не были зажжены. Луны тоже не было. Женщина стала переходить дорогу перед машиной. Как удалось водителю разглядеть ее?

(Так днем дело – то было)

Два человека играли в шашки. Каждый сыграл по пять партий и выграл по пять раз. Это возможно?

(Угу и проиграл тоже 5. В ничью играли. Также возможно, что они играли не друг с другом)

Что может быть больше слона и одновременно невесомым?

(Вакуум, но по объему он должен занимать много места)

Что все люди на Земле делают одновременно?

Что становится больше, если его поставить вверх ногами?

(Уровень песка в песочных часах)

Как спрыгнуть с десятиметровой лестницы и не ушибиться?

(Спрыгнуть с низщей ступеньки)

Что не имеет длины, глубины, ширины, высоты, а можно измерить?

(Кучу всего: скорость, время, работу, напряжение, IQ и тд)

Какой рукой лучше размешивать чай?

(В которой ложка, а если ложка есть в обоих, то которой удобней)

Когда сеть может вытянуть воду?

(Когда вода превратится в лед)

Hа какой вопрос нельзя ответить “Да” ?

(Ты умер? Ты, что глухонемой?)

Hа какой вопрос нельзя ответить “Hет” ?

(Вы живы? Пить будешь?)

Что имеет две руки, два крыла, два хвоста, три головы, три туловища и восемь ног?

((Инопланетянин;-) Или всадник на коне с соколом в руке)

Из какой посуды нельзя ничего поесть?

(Из пустой)

На березе росло 90 яблок. Подул сильный ветер, и 10 яблок упало. Сколько осталось?

(На березе яблоки не растут)

Если в 12 часов ночи идет дождь, то можно ли ожидать, что через 72 часа будет солнечная погода?

(Нет, через 72 часа будет снова полночь)

Под каким деревом сидит заяц, когда идет дождь?

(Под мокрым)

Можно ли из двух химических элементов создать еще один элемент?

(Да, если элемент гальванический)

Как известно, все исконно русские женские имена оканчиваются либо на “а”, либо на “я”: Анна, Мария, Ольга и т.д. Однако есть одно-единственное женское имя, которое не оканчивается ни на “а”, ни на “я”. Назовите его.

Назовите пять дней, не называя чисел (напр., 1, 2, 3,..) и названий дней (напр., понедельник, вторник, среда…).

(Позавчера, вчера, сегодня, завтра, послезавтра)

Когда черной кошке лучше всего пробраться в дом?

(Многие сразу говорят, что ночью. Все гораздо проще: когда дверь открыта)

На столе лежат линейка, карандаш, циркуль и резинка. На листе бумаги нужно начертить окружность. С чего начать?

(Надо достать лист бумаги)

Один поезд едет из Москвы в С.-Петербург с опозданием 10 минут, а другой – из С.-Петербурга в Москву с опозданием 20 минут. Какой из этих поездов будет ближе к Москве, когда они встретятся?

(В момент встречи они будут на одинаковом расстоянии от Москвы)

Из гнезда вылетели три ласточки. Какова вероятность того, что через 15 секунд они будут находиться в одной плоскости?

(100% , т.к. три точки всегда образуют одну плоскость)

На столе лежат две монеты, в сумме они дают 3 рубля. Одна из них – не 1 рубль. Какие это монеты?

(2 рубля и 1 рубль. Одна то не 1 рубль, а вот другая – 1 рубль)

С какой скоростью должна бежать собака, чтобы не слышать звона сковородки, привязанной к ее хвосту?

(Эта задача в компании сразу выявляет физика: физик сразу отвечает, что ей нужно бежать со сверхзвуковой скоростью. Разумеется, собаке достаточно стоять на месте)

Один оборот вокруг Земли спутник делает за 1 ч 40 минут, а другой – за 100 минут. Как это может быть?

(1 час и 40 минут = 100 минут)

Крыша одного дома не симметрична: один скат ее составляет с горизонталью угол 60 градусов, другой – угол 70 градусов. Предположим, что петух откладывает яйцо на гребень крыши. В какую сторону упадет яйцо – в сторону более пологого или крутого ската?

(Петухи не кладут яйца)

(Независимо от распределения жильцов по этажам, кнопка “1”)

Мальчик упал с 4 ступенек и сломал ногу. Сколько ног сломает мальчик, если упадет с 40 ступенек?

(Всего одну, т.к. вторая у него уже сломана, либо не больше ни одной, если повезёт;-))

Куда идет цыпленок, переходя дорогу?

(На другую сторону дороги)

Возможно ли такое: две головы, две руки и шесть ног, а в ходьбе только четыре?

(Да, это всадник на лошади)

Какое колесо не крутится при правом развороте?

(Запасное)

Еще одна загадка “с бородой”: шли два отца и два сына, нашли три апельсина. Стали делить – всем по одному досталось. Как это могло быть?

(Это были дед, отец и сын)

В комнате горело 50 свечей, 20 из них задули. Сколько останется?

(Останется 20: задутые свечи не сгорят полностью)

Какие слова изнуряли Винни-Пуха?

(Длинные и труднопроизносимые)

На край стола поставили жестяную банку, плотно закрытую крышкой, так, что 2/3 банки свисало со стола. Через некоторое время банка упала. Что было в банке?

(Кусок льда)

Как далеко в лес может забежать заяц?

Какое слово всегда звучит неверно?

(Слово “неверно”)

У трех трактористов есть брат Сергей, а у Сергея братьев нет. Может ли такое быть?

(Да, если трактористы – женщины, либо речь о разных Сергеях)

Что такое: входит сухим, выходит влажным, дарит тепло и радость?

(Пакетик с чаем)

Белый, а не сахар. Холодный, а не лёд.

В небе одна, в земле нету, а у бабы их целых две.

(Буква “Б”)

Сто одёжек и все без застёжек.

Что можно сбросить с обнажённой секретарши?

(Обнажённого начальника)

(Буква “А”)

Какое слово всегда звучит неверно?

(Слово “неверно”)

Кто под проливным дождём не намочит волосы?

Маленький, серенький, на слона похож.

(Слонёнок)

Может ли мужчина жениться на сестре своей вдовы?

Как хорошо тебе и мне, я под тобой, а ты на мне…

(Ёжик яблоко несёт)

(Боксеры могут морду набить)

Всем пиз@а, а одному езда.

(Муж в командировке)

Всем езда, а одному пиз@а. (Муж попал под троллейбус.)

*Всем пиз@а, никому езда. (Троллейбус упал с моста.)

Мальчик с девочкой в траве что-то делали на “Е”. (Ели землянику.)

Сколько требуется негров, чтобы похоронить человека? (Пять. Четверо несут гроб, а пятый идет спереди с магнитофоном.)

Около 40 млн. человек занимаются ЭТИМ по ночам. Что ЭТО такое(Internet.)

С горки ползком, в горку бегом. (Сопля.)

Стоит баба на полу, приоткрыв свою дыру. (Печка.)

Встанет, до небу достанет. (Радуга.)

В зубах доска, в глазах тоска. (Мужик провалился в деревенский толчок.)

Что нельзя съесть на завтрак? (Обед и ужин.)

*Без рук, без ног на бабу скок! (Коромысло.)

Прыгает ловко и ест морковку? (Бубка на диете.)

*Кто под проливным дождем не намочит волосы? (Лысый.)

Не петух, а поет, не дед, а бабку имеет, кто это? (Филипп Киркоров.)

*Одно колесо тысяча крыльев – что это? (Тачка с навозом.)

Что такое: твердое в мягкое вставляется, и шарики рядом болтаются?(Сеpьги.)

Две женщины у забора: одна приклеена, другая пришита… Что с ними нужно сделать? (Первую отодрать, вторую – отпороть.)

*Красный, длинный, 21? (Трамвай.)

*Что такое синее золото? (Любимая жена напилась.)

*Что возбуждается палочкой Коха? (1. туберкулез; 2. жена Коха.)

*Что объединяет горелый хлеб, утопленника и беременную женщину? (Не успели вытащить…)

*Два кольца, два конца… (Очень навороченный Новый Русский.)

В пустыне лежит мертвый мужчина. За плечами мешок, на поясе фляга с водой. На многие километры вокруг нет ни единой живой души. От чего умер человек и что в его мешке? (Человек умер от удара об землю, а в мешке – парашют, который не раскрылся.)

Чем отличается педагог от педофила? (Педофил по-настоящему любит детей.)

В 12-этажном доме есть лифт. На первом этаже живет всего 2 человека, от этажа к этажу количество жильцов увеличивается вдвое. Какая кнопка в лифте этого дома нажимается чаще других? (Независимо от распределения жильцов по этажам, кнопка “1”.)

С луком с яйцами, но не пирожок? (Робин Гуд.)

Едут в купе Буратино, Мальвина, честный таможенник и мент поганый. Играют в карты, в банке куча денег, поезд въезжает в тоннель. После выезда из тоннеля деньги исчезли. Кто украл деньги? (Мент поганый, т.к. первых троих в природе не существует…)

Откуда родом снежная баба? (Из ЗИМБАБве.)

Какая страна самая вооруженная? (Израиль,… там все с обрезами ходят.)

Что это такое – цвета сирени, назад видит так же, как и вперед, и прыгает выше колокольни? (Белая слепая лошадь, т.к. сирень бывает белая, а колокольня вообще не прыгает.)

Что такое: глаза боятся – руки делают. (Секс по телефону.)

Слово из трех букв, которого боится любой мужчина? (Еще!)

Что такое: самое доброе в мире приведение с моторчиком? (Запорожец.)

А любит Б, Б любит Ц?

Что делать А? (Найти другую Б.)

Что такое: голова есть, головы нет, голова есть, головы нет? (Хромой за забором.)

Похоронили проститутку, на надгробье написали: “Теперь они всегда будут вместе”. Кто они? (Ноги.)

Что это такое: летит и блестит? (Комар с золотым зубом.)

Стоп-бревно в случае экстренного торможения. (Столб.)

Что у женщины на теле, у еврея на уме, применяется в хоккее и на шахматной доске? (Комбинация.)

Какое слово из трех букв теперь чаще всего пишут на стенах туалетов в школах и вузах? (Сам ты Хуй! Правильный ответ – WWW!)

У какой социальной группы критические дни два раза в году? (Студенты.)

Идет ежик лысый – сколько ему лет? (18 – его в армию забирают.)

Висит груша – нельзя скушать. Почему?

(Боксеры могут морду набить.)

Мальчик с девочкой в траве что-то делали на “Е”.

(Ели землянику)

Сколько требуется негров, чтобы похоронить человека?

(Пять. Четверо несут гроб, а пятый идет спереди с магнитофоном.)

Около 40 млн. человек занимаются ЭТИМ по ночам. Что ЭТО такое?

С горки ползком, в горку бегом.

Стоит баба на полу, приоткрыв свою дыру.

Встанет, до небу достанет.

В зубах доска, в глазах тоска.

(Мужик провалился в деревенский толчок)

Что нельзя съесть на завтрак?

(Обед и ужин)

Без рук, без ног на бабу скок!

(Коромысло)

Прыгает ловко и ест морковку?

(Бубка на диете)

Кто под проливным дождем не намочит волосы?

Не петух, а поет, не дед, а бабку имеет, кто это?

(Филипп Киркоров)

Одно колесо тысяча крыльев – что это?

(Тачка с навозом)

Что такое: твердое в мягкое вставляется, и шарики рядом болтаются?

Две женщины у забора: одна приклеена, другая пришита… Что с ними нужно

(Первую отодрать, вторую – отпороть)

Красный, длинный, 21?

(Трамвай)

Что такое синее золото?

(Любимая жена напилась)

В пустыне лежит мертвый мужчина. За плечами мешок, на поясе фляга с водой. На многие километры вокруг нет ни единой живой души. От чего умер человек и что в его мешке?

(Человек умер от удара об землю, а в мешке – парашют, который не раскрылся)

Чем отличается педагог от педофила?

(Педофил по-настоящему любит детей)

В 12-этажном доме есть лифт. На первом этаже живет всего 2 человека, от этажа к этажу количество жильцов увеличивается вдвое. Какая кнопка в лифте этого дома нажимается чаще других?

(Независимо от распределения жильцов по этажам, кнопка “1”)

С луком с яйцами, но не пирожок?

(Робин Гуд)

Едут в купе Буратино, Мальвина, честный таможенник и мент поганый. Играют в карты, в банке куча денег, поезд въезжает в тоннель. После выезда из тоннеля деньги исчезли. Кто украл деньги?

(Мент поганый, т.к. первых троих в природе не существует…)

Откуда родом снежная баба?

(Из ЗИМБАБве)

Какая страна самая вооруженная?

(Израиль,… там все с обрезами ходят)

Что это такое – цвета сирени, назад видит так же, как и вперед, и прыгает выше колокольни?

(Белая слепая лошадь, т.к. сирень бывает белая, а колокольня вообще не прыгает)

Что такое: глаза боятся – руки делают.

(Секс по телефону)

Маленькое, желтенькое под кроватью лежит, на “З” начинается.

(Копейка. Почему на “З”? Закатилась…)

Что такое: голова есть, головы нет, голова есть, головы нет?

(Хромой за забором)

Похоронили проститутку, на надгробье написали: “Теперь они всегда будут вместе”. Кто они?

Как хорошо тебе и мне, я под тобой, а ты на мне.

(Ежик яблоко несет)

Что это такое: летит и блестит?

(Комар с золотым зубом)

Что такое: 90/60/90?

(Скорость при гаишнике)

Стоп-бревно в случае экстренного торможения.

Сережки для простаков.

Среднее арифметическое между велосипедом и мотоциклом?

Висит на стене, зеленое и пищит.

(Селедка. Висит на стене потому, что я ее туда повесил, зеленая потому, что я ее покрасил, а пищит, чтобы никто не догадался.)

Между ног болтается, воняет и орет?

(Мотоцикл)

Что у женщины на теле, у еврея на уме, применяется в хоккее и на шахматной доске?

(Комбинация)

На какой вопрос никто никогда не ответит “да”?

(Спящий на вопрос: “Вы спите?”)

Как можно ходить сидя?

(В туалете – на унитазе)

Когда человек бывает в комнате без головы?

(Когда высовывает ее из окна на улицу)

Какое слово из трех букв теперь чаще всего пишут на стенах туалетов в школах и вузах?

(Сам ты Х#@! Правильный ответ – WWW!)

У какой социальной группы критические дни два раза в году?

(Студенты)

Когда козе исполнится семь лет, что будет дальше?

(Пойдет восьмой)

Кругом вода, а посредине закон. Что это такое?

(Прокурор купается)

Может ли мужчина женится на сестре своей вдовы?

Почему шляпу носят?

(Потому что она сама не ходит)

Маленький, желтенький, в земле ковыряется.

(Вьетнамец мину ищет)

Маленький, желтенький в небе кувыркается.

По чему, когда захочешь спать, идешь на кровать?

Может ли страус назвать себя птицей?

(Нет, он не умеет разговаривать)

Что надо сделать, чтобы четыре парня остались в одном сапоге?

(Снять с каждого по сапогу)

Он от дедушки ушел, и от бабушки ушел…

Что это такое: власть лежит, а вода бежит?

(Депутату ставят клизму)

Что такое – зеленая, нажмешь кнопку – красная?

(Лягушка в миксере)

Косоглазый, маленький, в белой шубке, в валенках?

(Чукотский Дед-Мороз)

Что такое: падают с ветки золотые монетки?

(Обычное явление в стране дураков)

Сухой-клин, мокрый-блин?

(Мокрый клин, блин!)

Гаркнул гусь на всю Русь.

Что такое: два брюшка, четыре ушка?

(Кошачья свадьба)

Морщинистый Тит всю деревню веселит.

(Нехватка молодежи на селе)

Сколько яиц может удержать женщина в одной руке?

Что можно сбросить с обнаженной секретарши?

(Обнаженного начальника)

Что такое: ходит по стене и играет?

(Муха с плеером в ушах)

Когда женщина ногу поднимает, что видишь? Пять букв, на П начинается, на А кончается.

С какой скоростью должна бежать собака, чтобы не слышать звона сковородки, привязанной к ее хвосту?

(Собака должна стоять. Эта задача в компании сразу выявляет физика: физик отвечает, что ей нужно бежать со сверхзвуковой скоростью)

Идет ежик лысый – сколько ему лет?

(18 – его в армию забирают)

В две руки беру, между ног сую, пять минут потею, а потом балдею.

(Велотренажер)

Что ты смотришь на меня, раздевайся, я твоя.

(Кровать)

(Вариант: Вешалка)

Волосатая головка за щеку летает ловко.

(Зубная щетка)

Вокруг черное, посередине красное.

(Редиска в заднице у негра)

Вокруг черное, посередине белое.

(Редиска там же, только надкусанная)

На букву Х. называется, П. увидит подымается.

(Хобот пищу берет)

С когтями, а не птица, летит и матерится.

(Электромонтер)

То висячий то стоячий то холодный то горячий.

Ты помни его немножко, будет твердый, как картошка.

Маленький, серенький на слона похож.

(Слоненок)

Сто одежек и все без застежек.

Шел охотник мимо башни с часами. Достал ружье и выстрелил.

Куда он попал?

(В милицию)

Осенью питает, зимой согревает, весной веселит, летом холодит.

Куда идет цыпленок, переходя дорогу?

(На другую сторону дороги)

Мальчик упал с 4 ступенек и сломал ногу. Сколько ног сломает мальчик, если упадет с 40 ступенек?

(Всего одну, т.к. вторая у него уже сломана)

Что такое: маленькое лысенькое по лесу бежит?

(Ежик. Почему лысый? Из Чернобыля сбежал)

Не лает, не кусает, а в дом не пускает.

(Жена не пускает пьяного мужа)

Из какой посуды нельзя ничего поесть?

(Из пустой)

Четыре братца под одной крышей стоят.

Белый, а не сахар. Холодный, а не лед.

Какое слово всегда звучит неверно?

(Слово “неверно”)

Стоит Ивашка на одной ножке.

(Инвалид)

Почему поп шляпу покупает?

(Потому что даром не дают)

Под каким деревом сидит заяц, когда идет дождь?

(Под мокрым)

Что общего между деньгами и гробом?

(И то и другое сначала заколачивают, а потом спускают)

Два конца, два кольца, а по середине гвоздик.

(Жертва маньяка)

Какое колесо не крутится при правом развороте?

(Запасное)

Что такое: маленькое, черненькое, в стекло бьется?

(Младенец в духовке)

Сколько младенцев поместится в двухместную детскую коляску?

(А это, смотря как нарубить…)

Что такое: висит на стене и плачет?

(Альпинист)

Красная головка – работает ловко.

Что такое: сначала белый, потом вж-ж-жик, и красный?

(Пуделек соседки в миксере)

Без окон, без дверей, а внутри сидит еврей? Что это?

(Сара беременна)

Что такое: маленькая, зеленая, стоит на панели?

(Проститутка с другой планеты)

На веревке болтается, на “З” называется.

(Зоя Космодемьянская)

Кто быстрее доберется до холодильника – мышь или слон?

(Мышь. Она на велосипедике приедет)

Как узнать есть ли мышь в холодильнике, не открывая его?

(У холодильника должен стоять велосипедик)

Что такое: зеленое, лысое и скачет?

(Солдат на дискотеке)

Что это такое: синий, большой, с усами и полностью набит зайцами?

(Троллейбус)

Волоса, волоса…, а посредине колбаса.

(Кукуруза)

Маленький, желтенький, дверь собою открывает.

Чем отличается молодой холостяк от старого?

(Молодой холостяк прибирается в своем доме, чтобы пригласить женщину, а старый приглашает в дом женщину, чтобы она прибралась)

Зима, лес, все покрыто снегом. На большом обледеневшем пне лежит раздавленный член. Что это?

(На конец наступила зима)

Маленькая сморщенная, есть в каждой женщине.

(Изюминка)

Сколько программистов нужно, чтобы закрутить лампочку?

(Ни одного. Это аппаратная проблема, программисты их не решают)

Назовите слово, в котором 40 гласных.

(Сорока (сорок “А”))

В небе одна, В земле нету, А у бабы их целых две.

Чем кончаются день и ночь?

(Мягким знаком)

Мы ребята удалые, лезем в щели половые.

(Тараканы)

Что это такое: висит на стене и пахнет?

(Часы: в них кукушка сдохла)

Что такое – маленькое, беленькое кровь сосет?

Что такое – сидит на дереве, черное и каркает? На букву Ш.

(Ворона. Почему на Ш? Потому что шлангом прикинулась)

Что такое – маленькое, беленькое, летает и жужжит? На букву Б.

(Муха. Почему на Б? Потому что блондинка)

Тихо сзади подошел,

Дважды всунул и пошел.

(Тапочки)

Волос на волос, тело на тело – начинается темное дело.

(Глаз закрывается)

Что такое – 100 веревок и один х%й?

(Парашютист)

Что такое – 100х%ев и 100 веревок?

(Бурлаки на парашютах)

Висит – болтается, на три буквы называется. В середине “У”.

Что такое: Два конца, два кольца?

(Геевская свадьба)

Почему Ленин ходил в ботинках, а Сталин в сапогах?

(По земле)

Почему слоны не летают?

(По воздуху)

Чем человек отличается от паровоза?

(Паровоз сначала свистит, потом трогается, а человек сначала тронется, а потом ходит и свистит.)

Сверху черно, внутри красно.

Как засунешь – так прекрасно.

Туда-сюда-обратно:

Тебе и мне – приятно.

Разноцветное коромысло над рекою повисло.

(Признак начинающегося сумасшествия)

Чем отличается трактор от помидора?

(Помидор красный, а в тракторе дверь открывается наружу)

Что это: на окне сидит, по-французски говорит?

(Француз)

Чем их больше, тем вес меньше. Что это?

Что такое:

Черное – на одной ноге?

(Одноногий негр)

Из трех букв состоит,

На “Х” начинается,

Когда работает стоит,

Когда кончает, кланяется.

У какого молодца утром капает с конца?

(Самовар)

(Вариант: Водопроводный кран)

Кто такой: Сам не стреляет и другим не дает?

(Александр Матросов)

Висит груша – нельзя скушать.

(Чужая груша)

(Вариант: Тетя Груня повесилась)

Две черепахи (самец и самка) прогуливаются влюбленно вдоль берега держа друг друга за лапки. Через час обратно возвращается только самец. Где самка?

(Она осталась там – он забыл ее перевернуть)

Двое мужчин находятся на разных сторонах земли. Один переходит по канату через обрыв, а другому делает минет 70-летняя женщина. У обоих мужиков одна и та же мысль. Какая?

(Не смотреть вниз)

Ползет, ползет – камень съест. Снова ползет, ползет – камень съест.

Что это такое?

(Камнеежка)

Ползет, ползет – дерево съест. Снова ползет, ползет – дерево съест.

Что это такое?

(Камнеежка. Он и деревья ест)

В темной комнате, на белой простыне – два часа удовольствия.

(Киносеанс.)

Что у Адама спереди, а у Евы сзади?

(Буква “А”)

Почему в Париже девки ходят рыжие?

(По земле)

Две спины, одна голова, шесть ног. Что это такое?

(Человек на стуле)

Чем первый этаж отличается от девятого?

(С первого этажа упадешь: “Бух! – А-а!” А с девятого “А-а! – Бух!”)

Сам алый сахарный, кафтан зеленый бархатный.

(Негатив на цветной фотопленке, запечатлевший “нового русского”)

Что такое:

Черное – на двух ногах?

(Два одноногих негра)

Что такое:

Черное – на трех ногах?

Что такое:

Черное – на четырех ногах?

(Одноногий негр за роялем)

Интернет: Интернет и СМИ: Lenta.ru

Почти каждый день в сети появляются головоломки и картинки, ловко обманывающие зрение и ставящие пользователей в тупик. Они раз за разом вызывают ожесточенные споры, хотя порой решение лежит на поверхности, и для этого лишь нужно вспомнить азы школьной программы. «Лента.ру» отобрала самые обсуждаемые задачки и оптические иллюзии, наделавшие немало шума в интернете.

В начале апреля отец американского семейства убирал за своим сыном игрушечную железную дорогу. Обычно он просто забрасывал все элементы дороги в коробку, и на том уборка заканчивалась. Но на сей раз он решил разложить все детали по порядку и заметил интересную особенность — если положить две совершенно идентичные изогнутые части дороги друг под другом, то создается впечатление, что они разной длины.

Удивленный американец поинтересовался в Twitter, чем объяснить такой эффект. В результате его пост с видеозаписью ретвитнули несколько тысяч человек. Позже он сам нашел ответ на свой вопрос. Еще в 1982 году психолог Джозеф Джастроу показал, что никакой магии здесь нет: просто нижняя часть верхней фигуры короткая, а верхняя часть нижней — длинная, что и обманывает зрение.

В феврале 2016 года гражданка Канады предложила своим друзьям в Facebook поломать голову над детской задачкой про бананы, кокосы и яблоки. Головоломка представляет собой систему уравнений с несколькими неизвестными. Исходя из них, предлагается определить, чему равняется каждое из слагаемых.

Подвох кроется в финальном вычислении, где число фруктов меняется: бананов в связке становится на один меньше, а от расколотого кокоса остается половина. Это многих выбило из колеи. Одни пришли к выводу, что конечный вариант 14, у кого-то выходило 15, у третьих — 16. В результате только в Facebook запись собрала больше 180 тысяч перепостов и 2,7 миллиона комментариев.

Едва ли фотограф из Швейцарии Тицьяна Вергари, выкладывая в Instagram не самое шедевральное свое произведение, задумывалась, что снимок вызовет такой ажиотаж в сети. Она сфотографировала нескольких девочек у зеркальной стены, но сколько моделей приняло участие в съемке, не сообщила. Вот пользователи и ломают голову. Кому-то кажется, что девочек две, кто-то уверен, что четыре, есть версия и о том, что их было 12.

А это тест на остроту периферического зрения. В ярко-красном круге, напоминающем тот, что на японском флаге, предлагается разглядеть лошадь. Подавляющее большинство видит очертания животного, кому-то даже удается уловить детали: траву, гриву, седло. Но есть и такие, кому разглядеть лошадь не под силу.

Еще один тест на остроту периферического зрения. Гуляет по сети несколько лет. Если смотреть в середину изображения, можно заметить, как один из фиолетовых кружков в окружности становится салатовым.

Эту задачку на логику дети щелкают, как орехи, а взрослые впадают в замешательство.

Разгадка проста: речь идет о количестве букв в звуках, которые, как считается, издают приведенные на картинке животные: «гав», «му», «ку-ку», «кря», «бе», «хрю», «кукареку». Осел говорит «иа», поэтому правильный ответ — два. А у взрослых выходило и шесть, и восемь.

Так называемый финский армейский тест, больше известный как финский тест на гомосексуализм, позволяет определить, является новобранец геем или нет. Если в пятом круге человек не может разглядеть цифру шесть, то он — латентный гомосексуалист.

На самом деле этот тест — фейк. Еще в 2009 году его придумал блогер Фриц Морген. Он сообщил, что тест прислал его знакомый финн и приложил к нему проверочную черно-белую табличку, на которой действительно видны все шесть чисел. Морген хотел подшутить над теми, кто принимает все написанное в блогах за истину. И у него получилось: гетеросексуальные блогеры, сбитые с толку результатами теста, начали распространять его в своих аккаунтах. Правда, комментаторы быстро разоблачили подделку, но запись какое-то время даже успела повисеть в топе «Яндекса».

Этот тест возник после того, как отгремела история с «феноменом синего и белого платья». На картинках разного цвета изображены пилюли на ладони. Предлагается определить, какого они цвета.

Мнения разделились. Одни видят одинаково серые пилюли на обеих картинках, другие замечают разницу и считают, что одна из таблеток красного цвета, а другая — синего.

На самом деле обе таблетки серые, но мозг заставляет видеть их разными даже после того, как человек узнал правильный ответ. И действительно — всем объяснили, что пилюли серые, журналисты проверили это в фотошопе, но споры еще долго не утихали.

Спустя ровно год после белого и синего платья в Tumblr появилась новая задачка об одежде. Девушка, нарочито подчеркивая, что не хочет создавать новый мем, выложила фотографию олимпийки Adidas и поинтересовалась у своих подписчиков, какого она цвета.

Изображение: страница Poppunkblogger в Tumblr

Одни видят куртку в бело-голубых тонах, другие — в зелено-золотистых. Некоторые считают, что олимпийка — черно-коричневая. 67 процентов опрошенных «Лентой.ру» фолловеров в Twitter выбрали именно этот вариант.

Вероятнее всего, в случае с курткой, как и ранее с платьем, сработала оптическая иллюзия из-за неточной цветопередачи.

Вскоре автор, посетовав на ажиотаж вокруг ее записи, удалила сообщение, но перепостом публикации поделились десятки тысяч человек.

Загадки для 7 лет ~ Я happy МАМА

Загадки для возраста от 7 лет должны быть самыми разными: простыми, сложными, с юмором или с подвохом. В этом возрасте дети все больше стремятся изучать мир, но уже не только с помощью игры. Семилетки с удовольствием выполняют задания на логику, осваивают новые творческие техники, решают нестандартные задачки. Поэтому интересные загадки пригодятся родителям и педагогам для обучения и досуга маленького школьника.

Загадки для 7 лет

В нашей онлайн-подборке вы найдете самые лучшие загадки для детей 7 лет. Первоклассникам можно смело задавать сложные загадки, которые будут развивать их логическое мышление. Загадка пригодится вам и для веселой пятиминутки, и для викторины. Вместе со словесными играми, загадки можно использовать во время поездок или ожидания, чтобы отвлечь и приободрить ребенка.

В снежном поле по дороге
Мчится конь мой одноногий.
И на много-много лет
Оставляет чёрный след.

Карандаш

ОТВЕТ

Порой идут,
Порой стоят,
Порой лежат,
Порой висят.
А вот сидеть,
Скажу вам сразу,
Не приходилось им
Ни разу.

Часы

ОТВЕТ

Упадет – поскачет, а ударишь – не плачет

Мяч

ОТВЕТ

Течь — течёт,
А не вода.
Не кулак —
А бьёт
Всегда.

Электрический ток

ОТВЕТ

Он в лесу стоял, и никто его не брал,
В красной шапке модной, никуда не годный.

Мухомор

ОТВЕТ

Всегда во рту,
А не проглотишь.

Язык

ОТВЕТ

Как у Бабы у Яги
Нет совсем одной ноги,
Зато есть замечательный
Аппарат летательный.
Какой?

Ступа

ОТВЕТ

В школьной сумке я лежу,
Как ты учишься, скажу

Дневник

ОТВЕТ

Хоть у нас четыре ножки –
Мы не мышки и не кошки.
Хоть мы все имеем спинки–
Мы не овцы и не свинки.
Мы не кони, хоть на нас
Вы садитесь много раз.

Стулья

ОТВЕТ

Из небесной из реки
Разлетелись пузырьки
И на небе на ночном
Засверкали серебром.
Это ночью поздно
Появились …

Звезды

ОТВЕТ

Без крыльев по небу летает,
Расплачется и пропадает.

Туча

ОТВЕТ

Разноцветная дуга
Поднялась за облака,
Выше дома, выше горки,
Выше самой длинной елки.
Ярко под дождем сверкала,
А затем совсем пропала.
Что за странная дуга?
Это просто … !

Радуга

ОТВЕТ

Где вода столбом стоит?

В колодце

ОТВЕТ

По небу ходит
Маляр без кистей,
Краской коричневой
Красит людей.

Солнце

ОТВЕТ

Любят, с искренностью всей, в дом пускать они гостей.
Но в гостях, скажу вам сразу, сами не были ни разу.

Двери

ОТВЕТ

С неба — звездой,
На ладошку — водой

Снежинка

ОТВЕТ

Какая коровка, скажите, пока еще никому не дала молока?

Божья коровка

ОТВЕТ

Он летит издалека,
Гонит в небе облака,
Гонит волны в океане,
Кружит вихрем в урагане.
Может ласково подуть,
Может стихнуть и уснуть.
В каждом уголке на свете
Очень разный дует …

Ветер

ОТВЕТ

На окошке, а не кошка. Не еж, а в руки не возьмешь.

Кактус

ОТВЕТ

Без чего на белом свете
Взрослым не прожить и детям?
Кто поддержит вас, друзья?
Ваша дружная…

Семья

ОТВЕТ

В чистом поле, вдоль местечка, вьется змейкой, а не речка.

Тропинка

ОТВЕТ

В поле лестница лежит,
Дом по лестнице бежит.

Поезд

ОТВЕТ

Открыть свои тайны
Любому готова.
Но ты от неё
Не услышишь и слова.

Книга

ОТВЕТ

У тридцати двух воинов один командир

Зубы и язык

ОТВЕТ

С земли и ребенок поднимет, а через забор и силач не перекинет

Пух

ОТВЕТ

Вернуться в раздел Загадки для детей.

Сложные логические загадки. Какие насекомые одомашнены человеком? Взрослые загадки с подвохом

Развивать память и сообразительность вам помогут разные виды головоломок для взрослых – задачи на логику, сложные, смешные или заковыристые вопросы, разнообразные интеллектуальные математические игры.­

Игры и головоломки для взрослых на логику

Для решения разнообразных логических головоломок не требуется высокий уровень образования, начать их решать – полезное занятие абсолютно для всех.­ Разнообразные загадки с подвохом для взрослых тренируют нестандартное мышление, что поможет в повседневной жизни быстрее находить рациональные решения в непростых житейских ситуациях.

Загадки для взрослых­

Такой тип заданий позволит сразу же проверить правильность найденного вами решения. Чем хороши эти короткие загадки? Можно тематически, по ответам, сделать подборку для определенного праздника или застолья, чтобы развлечь гостей. В зависимости от состава приглашенных, хорошо подойдут загадки с подвохом, задания, которые рассмешат ваших друзей, или математические задания.

С подвохом

В заданиях с подвохом зачастую сам вопрос на первый взгляд выглядит нелогичным, например: на каком языке говорят молча? Когда же оглашается ответ, то первая реакция человека проявляется как несогласие с ним. На первый взгляд, вопрос и подобранная отгадка соединены непривычно и с двойным подтекстом. Но немного подумав, не можешь не согласиться, что такое дерзкое решение правильное и очень логичное (ответ: на языке жестов).

Смешные

Играть в разгадывание смешных загадок – одно удовольствие. Пока ваши гости будут высказывать варианты ответов на заковыристые вопросы, всю компанию гарантированно будут сотрясать взрывы хохота.­

Математические

В таких загадках надо угадать заданную цифру, либо высчитать результат, полагаясь не так на арифметику, как на сообразительность. Ответ, который выглядит очевидным и как будто лежит на поверхности, часто является неверным.

Интеллектуальные игры

Задачи на логику для взрослых – это многоходовые комбинации для тренировки мышления. Чтобы правильно их решить, надо продумывать действия на несколько шагов вперед. Такие задания относительно трудные, часто они бывают в виде оригинальных картинок, где нужно переставить или дорисовать некоторые элементы.

Конечно, родителям не всегда хочется утруждать драгоценное чадо и придумывать для него наисложнейшие загадки. Тем не менее такие вопросы, над ответом на которые нужно поразмыслить, полезны и нужны для детей и взрослых, независимо от возраста.

Зачем загадывать ребенку сложные загадки

Мамы и папы могут задаться вопросом о том, стоит ли вводить ребенка в заблуждение и включать в программу непростые задачки. Тем не менее, изучив информацию о том, насколько продуктивны наисложнейшие загадки для детей разных возрастов, родители тут же изменят свое былое мнение. Загадки на логику и с подвохом нужны по следующим причинам:

Это лишь некоторые факторы, которые указывают на то, что для детей обязательно нужны непростые вопросы, на которые надо найти ответы. Это поможет полноценно развиваться и быть грамотным.

Какими должны быть загадки

Понятное дело, что сложные загадки несколько отличаются от простых логических вопросов. Продумать программу развивающего занятия с такими заданиями следует предварительно, чтобы процесс прошел с легкостью и без заминок. Наисложнейшие загадки должны быть:

  • С подвохом.
  • Двусмысленны.
  • Такими, над ответом на которые стоит сильно задуматься.
  • Сложные загадки должны быть подобраны по возрасту ребенка. Это поможет мальчикам и девочкам найти ответы по уровню знаний. Отсюда следует, что малышам не стоит загадывать очень сложные загадки, для самых маленьких лучше подобрать вопросы с подвохом. Для деток же постарше можно выбирать такие вопросы, как для взрослых.

Стоит учитывать вышеуказанные факторы, подбирая логические вопросы для своего чада.

Загадки на логику для самых маленьких

Для деток дошкольного возраста можно взять во внимание следующие загадки:

На березе росло три яблока, а на тополе – пять груш, сколько всего фруктов на этих деревьях?

(Ни одного, на березе и тополе не растут фрукты)

Как можно найти черную кошку в темной комнате?

(Включить свет)

Каким станет красный с белой вышивкой платочек, если его опустить в Черное море?

Чего нельзя съесть на обед?

(Завтрак и ужин)

Что будет в следующем году с собакой, которой исполнилось пять лет?

(Ей будет шесть лет)

Чьи волосы не намокнут под проливным дождем?

(Лысого человека)

Как правильнее сказать: не видно белого желтка или не вижу белый желток?

(Никак, желток не бывает белым)

Утка, стоящая на одной лапе, весит три килограмма, сколько будет весить эта же утка, если она станет на две лапы.

(3 килограмма)

Два яйца варятся 4 минуты, сколько будут вариться десять яиц?

(4 минуты)

Возле лавочки отдыхает кошка. И хвост, и глаза, и усы – все, как у кошки, но это не кошка. Кто отдыхает возле лавочки?

Угадай, что пропадет, если съесть бублик?

Как можно зажечь спичку, когда находишься под водой?

(Можно, если находишься в подводной лодке)

В зале было зажжено 30 свечей. Зайдя в комнату человек, погасил 15 из них. Сколько свечей осталось в зале?

(Осталось 30 свечей, затушенные свечи по-прежнему в комнате)

У дома неровная крыша. Одна сторона опущена больше, другая меньше. Петух сел на вершину крыши и снес яйцо, в какую сторону оно покатится?

(Никуда не покатится, петух не несет яйца)

Под какое дерево прячется лисица во время дождя?

(Под мокрое)

На каких полях не растет ни одного растения?

(На полях шляпки)

Такие сложные загадки на логику для самых маленьких вызовут водоворот эмоций и интереса. Самое главное, давать ребенку намеки, благодаря которым он сможет найти правильный ответ.

Сложные загадки с подвохом для школьников

Детям школьного возраста можно подобрать вопросы еще труднее. Очень сложные могут быть следующими:

Ты на соревновании по бегу. Когда ты перегнал того, кто бежал последним, каким ты стал?

(Такого не может быть, ведь последнего бегуна нельзя перегнать, потому что он последний и сзади него не может быть еще кого-то)

У троих владельцев автомобилей был брат Алеша. Но у Алеши не было ни одного брата, как такое возможно?

(Возможно, если у Алеши были сестры)

Каким вы станете по счету, если перегнали второго по очереди бегуна?

(Многие ответят первым, но это неправильно, потому что перегнав второго бегуна человек станет вторым)

Такие сложные загадки с подвохом непременно понравятся школьникам. Поразмыслив над ответом, его будет несложно озвучить.

Взрослые загадки с подвохом

Порой взрослые люди, словно дети. Поэтому очень сложные загадки им тоже придутся по душе. Людям старше школьного возраста можно задать следующие логические вопросы:

Едет трамвай с пятью пассажирами. На первой остановке вышли два пассажира, зашли четыре. На следующей остановке никто не вышел, зашли десять пассажиров. Еще на одной станции вошли пятеро пассажиров, один вышел. На следующей – вышли семеро, зашли восемь человек. Когда была еще одна остановка, вышли пятеро и никто не заходил. Сколько всего остановок было у трамвая?

(Ответ в данной загадке не столь важен. Суть в том, что все участники, скорее всего, будут считать количество пассажиров и вряд ли кто-то примет решение подсчитывать остановки)

В дверь звонят. Ты знаешь, что за ней находятся твои родственники. В твоем холодильнике стоят шампанское, холодная вода и сок. Что ты откроешь в первую очередь?

(Дверь, потому что гостей сначала нужно впустить в квартиру)

Здорового человека, который не болеет, не имеет инвалидности и у которого все в порядке с ногами, выносят на руках из больницы. Кто это?

(Новорожденный ребенок)

Зашел ты в комнату. В ней находятся пять котов, четыре собаки, три попугая, две морские свинки и жираф. Какое количество ног стоит в комнате на полу?

(На полу две ноги. У животных лапы, ноги только у человека)

Трое заключенных, не зная запланировали сбежать из тюрьмы. Тюрьму окружала речка. Когда сбегал первый заключенный, на него напала акула и съела его. Так погиб первый из сбегающих. Когда второй заключенный осуществлял попытку бедствия, его заметили часовые и за волосы потащили на территорию тюрьмы, там его расстреляли. Третий заключенный убежал нормально, и больше его никто не видел. Что не так в этой истории?

(Акул в речке не бывает, заключенного не могли потащить за волосы, потому что их бреют наголо)

Такие загадки понравятся взрослым участникам мероприятия.

Как мотивировать ребенка к участию в развивающем занятии

Понятное дело, что детям обязательно нужна мотивация для того, чтобы участие в игре было азартным и желаемым. Достаточно просто пообещать ребеночку какой-то презент и, конечно же, вручить его в конце игры.

/ 10 загадок на логику для определения профпригодности на роль следователя.

№1-я загадка
Парень встречается с девушкой, они любят друг друга. В субботу вечером они должны увидеться, девушка готовится к встрече. За час до назначенного времени приходит смс: “Извини, дорогая, увидеться не сможем. Я у родителей, к ним приехали гости, будут допоздна, придется остаться там на ночь. Встретимся завтра, целую, люблю”.Девушка понимает, что парень ей врет. Как она поняла?

№2-я загадка
На входе в хранилище банка стоит охранник. Рядом притаился шпион. Подходит к охраннику один служащий банка, охранник ему: “Шесть”. Служащий в ответ: “Пять”. Охранник пропустил. Следующий подходит – охранник:”Пять”, служащий:”Четыре”. Охранник пропустил. Рискнул шпион, подходит – охранник:”Четыре”, шпион:”Три”. Охранник с воплями:”Неправильно, попался, гад!” скрутил шпиона.Вопрос: почему неправильно и каков правильный ответ.

№3-я загадка
Несколько фирм занимались продажей товаров, используя для этих целей специально подготовленные каталоги. Так как товары у фирм были почти одинаковые, то и контингент клиентов у них был одинаков, так же как и их количество. Вскоре одна из фирм изменила свой каталог. На первый взгляд казалось, что от этих изменений он только проиграл и стал менее востребованным. Однако, после изменения продажи у этой фирмы пошли лучше, чем у конкурентов. Какое изменение они внесли и как оно действовало?

№4-я загадка
Собираясь в 30-ти дневный отпуск врач дал пациенту 30 таблеток лекарственного средства “A” и 30 таблеток средства “B”. Принимать эти лекарства нужно каждый день все 30 дней и строго по одной таблетке каждого средства в день, иначе неизбежен летальный исход. Таблетки внешне абсолютно одинаковы, вес, плотность и т. п. также полностью совпадают. Однажды пациент достал из одной упаковки 1 таблетку, а из другой выронил 2, и все 3 таблетки перепутались. Как соблюсти инструкции врача?

№5-я загадка
Он формой как луна, а как погода – разный.Имеет он закат, как солнце и рассвет.Другой нас заблуждает признак важный:- В себе способен он “вместить” любой предмет.

№6-я загадка
Человек, покупающий это, сам этим не пользуется.Человек, производящий это, производит это не для себя.Человек, пользующийся этим, об этом не знает.Что это?

№7-я загадка
В комнате находятся три выключателя, каждый из которых соединен с одной лампой. Найдите способ определить пару “выключатель – лампочка”. Если лампочки находятся в другой комнате и зайти в нее можно лишь один раз.

№8-я загадка
Женщина живёт на 12-м этаже здания. Каждое утро, отправляясь на работу, она вызывает лифт на 12-й этаж и спускается в нём до первого этажа. Но вечером, возращаяясь с работы, она доезжает на лифте только до 7-го этажа, а потом, чтобы добраться до своей квартиры, преодолевает ещё 5 этажей пешком. Почему?

№9-я загадка
Юрий и Татьяна дружат с детства. У Юры братьев и сестер поровну. У Татьяны сестер втрое больше, чем у Юрия, а всего столько, сколько детей у родителей Юры.Сколько сестер у Татьяны и братьев с сестрами у Юрия?

№10-я загадка
Вася, Петя и Коля живут по соседству в доме на Еловой улице в 15, 17 и 19 квартирах. У каждого из них особая дверь – одна покрашена в красный цвет, другая в синий, а третья в темно-зеленый.Петина дверь синяя. Он работает слесарем и дружит с Васей, который живет в 15 квартире. Ни Вася, ни Петя не ладят с Колей. В 17 квартире – красная дверь.Определите, где живет каждый из них и какого цвета его дверь.

№ 1 Ответ:
Если он действительно у родителей, то должен был написать “остаться здесь на ночь”

№ 2 Ответ:
В слове “шесть” – пять букв, в слове “пять” – четыре, в слове “четыре” – шесть.
Ответ – шесть.

№ 3 Ответ:
Они стали выпускать свой каталог меньшим по размеру, чем каталоги других фирм, из-за чего этот каталог всегда надо было класть самым верхним в стопке

№4 Ответ:
Нужно из первой упаковки достать еще одну таблетку – итого 4. Все порезать пополам. В один день принять 4 левых половинки, в другой – 4 правых. Главное запомнить, где половинка от какой таблетки.
или
Добавить ещё одну таблетку, размолоть до порошка, смешать, и разделить на две части

№5 Ответ:
ГЛАЗ
P.S. Если вы посмотрите в глаз человека, смотрящего на луну, вы увидите там луну.

№6 Ответ:
Это гроб.

№ 7 Ответ:
Нужно включить два выключателя. Через какое-то время один выключить. Зайти в комнату. Одна лампочка будет горящая от включенного выключателя, вторая горячая – от включенного и выключенного, третья – холодная, от нетронутого выключателя.

№8 Ответ:
Эта женщина очень маленького роста: ей удаётся дотянуться только до кнопки 7-го этажа в кабине лифта.

№9 Ответ:
У Юрия 1 брат и 1 сестра. У Татьяны 3 сестры

№10 Ответ:
Вася живет в 15 номере за темно-зеленой дверью. Коля живет в 17 номере, за красной дверью. Петя живет в 19 номере, за синей дверью

И маленькие детишки, и ребята школьного возраста безумно любят совместные игры с родителями, бабушками или дедушками. Поэтому интересные загадки с ответами обязательно привлекут их внимание. Самое главное, чтобы взрослые продумали сценарий, по которому будет проходить увлекательная игра.

Загадка как способ развития ребенка

Вообще, интересные с ответами – не просто вдохновляющая игра. Это способ в развлекательной форме развить:

  • мышление;
  • логику;
  • фантазию;
  • усидчивость;
  • стремление.

Это лишь некоторые факторы, указывающие на то, что сложные интересные загадки с ответами – это не только весело, но и полезно для чада.

Увлекательная игра с логическим уклоном

Конечно же, лучше всего перевести задания в игровую форму. Это можно сделать с учетом того:

  • сколько детей принимает участие в мероприятии;
  • какого возраста ребята;
  • в чем задача игры.

Можно эстафету, в которой каждый ребенок сможет проявить смекалку и скорость мышления. Интереснее будет, если деткам вручать монетки за каждый правильный ответ. Потом, в конце игры, можно монетки обменять на какую-то сладость или игрушку. В игровой форме детки не будут воспринимать задание как урок, поэтому его будет выполнять гораздо интереснее и веселее.

Самые интересные загадки с ответами на логику

Задачки на мышление помогут проверить, насколько ребенок умеет мыслить нестандартно. Именно для этой цели понадобятся интересные загадки с ответами.

В комнате три дивана, у каждого из них по четыре ножки. Еще в комнате пять собак, у каждой из них по четыре лапы. Позже зашел в комнату человек. Сколько ног в комнате стало?

(Две, у дивана – не ноги, а у животных – лапы.)

Меня зовут Витя, мою младшую сестру – Алена, среднюю – Ира, а старшую – Катя. Как зовут брата каждой из сестер?

Какое колесо автомобиля не шевелится во время правого поворота?

(Запасное.)

Где оказался великий путешественник Геннадий, когда свеча в его руках потухла?

(В темноте.)

Ходят, а с места ни на шаг.

Два друга играли в футбол три часа. По сколько времени играл каждый из них?

(По три часа.)

Как называется слон, у которого нет хобота?

(Шахматный.)

Шла девочка Арина в сторону дачи и несла в корзине яблочные пирожки. Навстречу шли Петя, Гриша, Тимофей и Семен. Сколько всего детей шло на дачу?

(Только Арина.)

Что постоянно становится больше, а меньше никогда?

(Возраст.)

Бабушка несла продавать двести куриных яиц. По пути у нее оторвалось дно пакета. Сколько яиц она донесет до рынка?

(Ни одного, из порвавшегося дна все выпали.)

Логические интересные загадки с ответами понравятся детям. Взрослые также с огромным удовольствием будут обдумывать такие вопросы.

Увлекательные и интересные загадки с хитрым ответом

Также стоит обратить внимание на задания, разгадки в которых совершенно непредсказуемы. Интересные загадки с ответами представлены вашему вниманию ниже.

Какой станет зеленая футболка, если в ней зайти в Черное море?

Животное, которое есть в зоопарке, а также на пешеходной зоне трассы.

Горят два дома. Один дом богатых людей, а другой – бедных. Какой из домов первым потушит скорая помощь?

(Скорая помощь не тушит пожары.)

Сколько лет в году?

(Одно лето.)

Его можно завязать, но нельзя развязать.

(Разговор.)

Перед кем даже короли и лорды снимают шляпы?

(Парикмахер.)

В вагоне метро ехало пятнадцать человек. На одной остановке вышло трое, а зашло пятеро. На следующей остановке никто не выходил, а зашло три человека. На еще одной остановке вышло десять человек, а зашло пятеро. Еще на одной остановке вышло семь человек, зашло трое. Сколько всего остановок было?

Река, которая есть даже во рту у человека.

Муж жене подарил колечко и сказал: “Я уезжаю работать за границу. Когда я уеду, посмотри, что написано на внутренней стороне украшения”. Когда жене было весело, она читала надпись, и ей становилось грустно, а когда ей было грустно, то надпись дарила силы. Что было написано на кольце?

(Все пройдет.)

Что ты можешь взять в левую руку, но никогда не сможешь этого взять правой рукой?

(Правый локоть.)

Вот такие интересные загадки с ответами помогут малышу пошевелить извилинами и хорошенько подумать.

Загадки на логику для самых маленьких

Самым маленьким детишкам лучше всего предлагать к разгадыванию самые несложные для восприятия задачки.

В саду на елке выросло пять яблок, а на березе – четыре груши. Сколько всего плодов?

(Нисколько, на этих деревьях фрукты не растут.)

Из какой тарелки ты ничего не сможешь скушать?

(Из пустой.)

В вазочке четыре ромашки, три розы, два тюльпана и две хризантемы. Сколько всего ромашек в вазе?

(Четыре ромашки.)

Витя сделал три горочки песка. Потом он их все соединил в одну и добавил еще одну собранную горочку. Сколько всего горок получилось?

Пришел декабрь, в саду у бабушки поспели вишни и малинка. Сколько деревьев или кустов подарили плоды?

(Ни одного, в декабре плоды не растут.)

Две сестры-близняшки Аня и Таня решили устроить игру и во время отпуска договорились, что одна будет говорить только правду, а вторая всегда будет говорить неправду. Девочки со двора придумали, как вычислить, кто из них врет. Какой вопрос они задали?

(А солнце светит?)

В снегу она одна, в морозе ее нет, а в сосиске их целых три. Что это?

(Буква «С».)

Какой человек не намочит волосы даже под ливнем?

Может ли павлин сказать, что он птица?

(Нет, потому что павлины не разговаривают.)

Два мальчика залезли на чердак, чтобы найти там старые игрушки. Когда они вышли на солнечный свет, было видно, что у одного лицо все запачканное, а у второго чистое. Умываться первым пошел тот мальчик, лицо которого чистое. Почему?

(Он увидел, что второй грязный, и подумал, что тоже такой.)

Сколько йогуртов ты можешь скушать на пустой желудок?

(Один, остальные не на голодный желудок.)

Насколько быстро должна бежать кошка, чтобы банка, привязанная к хвосту, не шумела?

(Кошка должна сидеть на месте.)

Логические загадки для школьников

Мальчишкам и девчонкам, которые посещают школу, стоит задавать задачки посложнее, где нужно хорошенько подумать. Давайте рассмотрим, какие детские интересные загадки с ответами можно включить в развлекательное мероприятие.

Как можно прыгнуть с двадцатиметровой лестницы и не удариться?

(Прыгнуть с нижних ступенек.)

У собаки на шее была двенадцатиметровая цепь. Она прошла более двухсот метров. Как так получилось?

(Она не была привязана.)

Что делать, если ты видишь зеленого человечка?

(Переходить пешеходный переход.)

Может ли в комнате человек находиться без головы?

(Да, если высунул голову в форточку или окно.)

Можно ли увидеть прошлогодний снег? Когда?

Когда белой кошке будет удобнее попасть в темную комнату?

(Когда открыта дверь.)

У тебя в руках спичка, в темной комнате при входе стоит свеча и плита. Что зажжешь в первую очередь?

Что весит больше – один килограмм сладкой ваты или один килограмм железных гвоздей?

(Весят одинаково.)

Сколько крупинок гречки войдет в стакан?

(Нисколько, крупинки не ходят.)

У каждой из четырех родных сестер Анжелы, Кристины, Ольги и Ирины есть по одному брату. Сколько в семье детей?

Пришла в больницу на обследование. Она была сестрой доктора, но доктор не был ее братом. Кем был врач?

(Сестрой.)

Настя и Алиса играли игрушками. Одна из девочек играла с плюшевым мишкой, а другая – с машинкой. Настя не играла с машинкой. Какая игрушка была у каждой девочки?

(Настя – с мишкой, а Алиса – с машинкой.)

Сколько углов станет у прямоугольного стола, если отпилить один угол?

(Пять углов.)

Настя и Кристина вдвоем пробежали восемь километров. По сколько километров пробежала каждая девочка?

(По восемь каждая.)

Эти очень интересные загадки с ответами помогут ребенку показать и умственные способности. Родителям стоит проявить фантазию и устроить настоящий марафон эмоций.

Почему следует загадывать загадки

Совместное времяпровождение очень нужно малышу, чтобы он понимал, как родители его любят. Поэтому следует чаще устраивать такие мероприятия. Ребенок также сможет проявить свои таланты в процессе игры.

Веселый праздник

Мамам, папам, бабушкам и дедушкам следует понимать, что чем ярче будет событие, тем интереснее и веселее будет ребенку. Поэтому стоит:

  • устроить карнавал, в котором все будут в красивых костюмах;
  • придумать подарки для победителя эстафеты;
  • награждать того, кто набрал максимум очков, какими-то презентами за каждый правильный ответ.

Детишки будут рады любому событию. А когда обычный вечер превращается в праздник, то счастья не будет предела. Все зависит от фантазии и идей родителей. Радуйте своих маленьких сыночков и доченек, и они отблагодарят вас блеском в глазках и довольными улыбками.

Загадки на смекалку: 40 головоломок на логику ✍

Ниже вы найдете загадки на логику с ответами с подвохом. Выбирайте самые сложные и пошлые головоломки для взрослых и детей. Каждая логическая задача вызовет интерес любого человека, как и загадки для взрослых. Прочитайте их вместе со своими друзьями. Все эти логические загадки с ответами понравятся им.

  1. Не лёд, а тает, не лодка, а уплывает. (Зарплата).
  2. Эти три телезвезды давно на экране. Одного зовут Степан, второго — Филипп. Как зовут третьего? (Хрюша).
  3. Что нужно делать, когда видишь зеленого человечка? (Переходить улицу).
  4. Это дается нам трижды. Первые два раза бесплатно. А вот за третий придется заплатить. (Зубы).
  5. Какое слово начинается с трех букв «Г» и заканчивается тремя буквами «Я»? (Тригонометрия).
  6. Что может в одно и то же время: стоять и ходить, висеть и стоять, ходить и лежать? (Часы).
  7. Гуси шли на водопой гуськом (один вслед за другим).
    Один гусь посмотрел вперёд – перед ним 17 голов. Посмотрел назад – за ним 42 лапы.
    Сколько гусей шло на водопой? (17 впереди, 21 позади, да и сам гусь, который головой вертел).
  8. – Она красная?
    – Нет, черная.
    – А почему она сейчас белая?
    – Потому, что еще зеленая.
    О чем речь? (Черная смородина).
  9. Ты ‒ летчик самолета, летящего из Лондона в Берлин с двумя пересадками в Париже. Вопрос: как фамилия летчика? (Твоя фамилия (в начале загадки «ты ‒ летчик…»).
  10. Ты входишь в темную комнату. В комнате есть газовая плита, керосиновая лампа и свечка. У тебя в кармане коробок с 1 спичкой. Вопрос: что ты зажжешь в первую очередь? (Спичку).
  11. Бизнесмен купил лошадь за 10$, продал ее за 20$ Потом он купил ту же самую лошадь за 30$, а продал за 40$. Вопрос: какова суммарная прибыль бизнесмена от этих двух сделок? (20$).
  12. В лесу заяц. Пошел дождь. Вопрос: под каким деревом заяц спрячется? (Под мокрым).
  13. Кто утром ходит на 4 ногах, днем на 2-х, а вечером на 3-х? (Человек. В младенчестве на четвереньках, потом на двух, потом с палочкой).
  14. Шёл сильный ливень. По дороге ехал автобус. Все люди в автобусе спали только водитель не спал. Вопрос: как звали водителя и какой номерной знак автобуса? (Из-за ливня номер автобуса не разглядеть, а водитель Толя (только(а) ‒ Толька).
  15. Друг другу на встречу идут 2 человека. Оба совершенно одинаковые. Вопрос: кто из них первый поздоровается? (Более вежливый).
  16. Собака-3, кошка-3, ослик-2, рыбка-0. Чему равняется петушок? И почему? (Петушок-8 (кука-ре-ку!), собака-3 (гав), кошка-3 (мяу), ослик-2 (иа), рыбка-0 (не издает звуки).
  17. В 12-этажном доме есть лифт. На первом этаже живет всего 2 людей, от этажа к этажу количество жителей увеличивается вдвое. На каком этаже в этом доме чаще всего нажимается кнопка вызова лифта? (На первом этаже, независимо от распределения жителей по этажам).
  18. Крестьянину необходимо перевести через реку волка, козла и капусту. Лодка такая маленькая, что в ней кроме крестьянина может поместиться еще только один (пассажир). Но если оставить волка с козлом, то волк его съест, если оставить козла з капустою, то будет съедена капуста. Как быть крестьянину? (Переправу нужно начинать с перевозки козла. Потом крестьянин возвращается и берет волка, которого перевозит на другой берег и там его оставляет, но берет назад на первый берег козла. Тут он его оставляет и перевозит к волку капусту. А потом, вернувшись, перевозит козла).
  19. Что поднимает вас наверх и опускает вниз, при этом постоянно находясь на одном и том же месте? (Эскалатор).
  20. Бочка с водой весит 50 килограмм, что нужно добавить, чтобы она стала весить 15 килограмм? (Дырку).
  21. Как вы думаете, каких камней не бывает в речке? (Сухих).
  22. Как вы думаете, какой рукой лучше всего размешивать кофе со сливками и с сахаром? (Той рукой, в которой ложка).
  23. Скажите, что можно удерживать, не касаясь его руками? (Свое дыхание).
  24. Человек попал под дождь, и ему нигде и ничем было укрыться. Домой он пришёл весь мокрый, но не один волос на его голове не промок. Почему? (Он был лысым).
  25. Какое слово всегда звучит неверно? (Слово «неверно»).
  26. Два рога ‒ не бык, шесть ног без копыт, когда летит — воет, сядет — землю роет. (Жук).
  27. На край стола поставили металлическую банку, плотно закрытую крышкой, так, что 2/3 банки свисало со стола. Через некоторое время банка упала. Что было в банке? (Кусок льда).
  28. Электропоезд идет по ветру. Куда идет дым? (У электропоезда нет дыма).
  29. Почему белые медведи не едят пингвинов? (Медведи живут на Северном полюсе, а пингвины на южном полюсе).
  30. Когда курица стоит на одной ноге, она весит 2 кг. Сколько она будет весить, если встанет на две ноги? (2 кг).
  31. Одно яйцо варится 3 минуты. Сколько будут варится 2 яйца? (3 минуты).
  32. Когда небо ниже от земли? (Когда смотришь в воду).
  33. Что не может войти даже в самую большую кастрюлю? (Её крышка).
  34. Какие зубы у человека появляются последними? (Искусственные).
  35. Почему кукушка не вьет гнезд? (Потому что живет в часах).
  36. Что нужно делать, когда видишь зеленого человечка? (Переходить улицу).
  37. Как вы думаете, возможно ли такое: «позади меня лошадь, впереди меня самолет, а сам я на корабле»? (Такое возможно на карусели).
  38. Что получится, если барабан разделить на барабан? (Барабанная дробь).
  39. Оно в комнате, а комната в нем. Что это? (Зеркало).
  40. Больше часа, меньше минуты. Что это? (Секундная стрелка на часах).

Вы прочитали все детские интересные загадки с подвохом с ответами смешные. Теперь сможете выбрать одну из них и рассказать ученикам своего класса. Все вопросы и логические задачи для взрослых ру с ответами очень интересные и предназначены также для детей старше 7 лет.

Загрузка…

комментарии

Круто! Оссобенно для квестов

добавить комментарий

7 вопросов с подвохом –

Тренируем логику и английский 😉

Предлагаем вам загадки на английском со словариком. Эти семь подковыристых вопросов с переводом помогут потренировать логическое мышление  и английский язык. Под каждым вопросом мы будем давать полезные объяснения относительно лексики или грамматики.

Начнем 🙂

(P.S. вопросы не такие уж и сложные, поэтому прежде, чем давать ответ, постарайтесь догадаться сами)

Ты мой брат, но я не твой брат. Кто я?

На данном примере мы предлагаем разобрать, когда употребляется your и my, а когда — yours и mine. Ведь несмотря на то, что правило довольно простое, учащиеся нередко путаются в этих слов и употребляют их неправильно. Первую пару слов —  your и my — мы используем, когда после у нас идет существительное. Например:

This is my pen. — Это моя ручка.

Вторая пара слов  — yours и mine — употребляется, когда после слов нет существительного. Например:

This pen is mine. — Эта ручка моя.

(Ответ: я твоя сестра.)

62-летняя женщина и 16-летняя девушка заходят в ресторан. Мимо них прошел мужчина, и обе женщины сказали «Здравствуй отец». Как такое возможно?

(Ответ: мужчина был священником.)

А теперь к лексике:

Walk past — проходить мимо;

Priest |priːst| — священник. Обратите внимание на транскрипцию — «i» в этом слове является долгой.

Какое слово становится короче, когда вы добавляете к нему две буквы?

Подсказка: перевод дан просто для понимания сути, для ответа ориентируйтесь на вопрос на английском.

(Ответ: слово «shorter»)

Обратите внимание, что в данном предложении используется вопросительное слово «what», а не «which». Многие люди, изучающие английский язык, путаются, какое из этих слов в каких случаях употреблять. Запоминаем: which мы употребляем в том случае, когда у нас есть выбор между несколькими вариантами. Например:

Which dress is more suitable for the dinner with my colleagues — my brown one or my black one?

Какое платье больше подходит для ужина с коллегами — коричневое или черное?

What употребляется, когда мы не выбираем между чем-то, а когда вы вообще не владеете информацией или выбираете один вариант из всех возможных. Например:

What dress should I wear for the dinner  with my colleagues?

Какое платье стоит мне надеть на ужин с коллегами?

В данном случае девушка не пытается сделать выбор между двумя платьями, а думает, какое платье ей вообще следует надеть на ужин.

Мужчина гулял по улице в сильный ливень в течение 30 минут, при этом, ни один волосок на его голове не намок. У него не было шапки, он не нес зонтик и не держал ничего у себя над головой. Его одежда промокла насквозь. Как такое могло случиться?

Полезная лексика:

Rainstorm |ˈreɪnstɔːrm| — ливень;

To get soaking |ˈsoʊkɪŋ| wet — промокать насквозь.

(Ответ:  мужчина был лысый — bald |bɔːld|)

Две девочки родились от той же матери, в тот же день, в то же время, в том же месяце, в тот же год, и они не двойняшки. Как такое может быть?

Обратите внимание на предлоги в данном предложении. Родиться от одной матери — to be born TO the same mother (а не «from»). Также данное предложение — отличный наглядный пример предлогов времени:

со днем мы используем ON — on the same day;

когда речь идет о конкретном времени, используется AT — at the samer time;

с годом и месяцем употребляется IN — in the same month and (in the same) year.

(Ответ:  это две девочки из тройни).

Словарик: twins — близнецы, двойня; triplets — тройня)

Где вы найдете дороги без транспорта. леса без деревьев и города без домов?

Полезная лексика:

Road |roʊd|- дорога;

Vehicle |ˈviːəkl| — автомобиль, сухопутный транспорт.

Forest |ˈfɔːrɪst|  — лес.

(Ответ:  на карте — on a map).

Девочке было тринадцать на прошлый день рождения, и будет 15 на следующий. Как такое возможно?

В этом предложении стоит опять обратить внимание на предлог, который употребляется перед названием дня — ON. Праздники мы используем с этим предлогом: on Christmas, on Birthday, on Easter. Однако стоит учесть, что в этом случае мы будем подразумевать именно сам праздничный день. Если мы имеем ввиду праздничный период, например, говоря о Рождестве или Пасхе подразумеваем не только день самого праздника, а период времени (к примеру, три выходных на Пасху), — мы будем употреблять предлог at: at Christmas, at Easter.

(Ответ:  сегодня у нее День рождения).

Надеемся, вам было интересно отвечать на эти загадки на английском:)

31 сложная математическая загадка и задача со словами для будущих гениев

Загадка: Если есть 4 яблока и вы убираете 3, сколько у вас есть?
Ответ: Вы взяли 3 яблока, так что у вас 3!

Загадка: 300-футовый поезд движется со скоростью 300 футов в минуту, должен пройти через 300-футовый туннель. Сколько времени займет поездка поезда по туннелю?
Ответ: Две минуты. У передней части поезда уходит одна минута, а остальной части поезда потребуется две минуты, чтобы очистить туннель.

Загадка: Мобильный телефон и чехол для телефона в сумме стоят 110 долларов. Сотовый телефон стоит на 100 долларов дороже чехла для телефона. Сколько стоил мобильный телефон?
Ответ: 105 долларов (не 110 долларов)

Загадка: Роберт и Дэвид сыграли несколько матчей в гольф друг против друга за неделю. На каждом матче играли за пиццу, но до конца недели пиццы не покупали. Если в какой-то момент Роберт и Дэвид выигрывали одинаковое количество побед, эти пиццы отменялись.Роберт выиграл четыре матча (но без пиццы), а Дэвид выиграл три пиццы. Сколько раундов в гольф было сыграно?
Ответ: Eleven
Пояснение: Дэвид выиграл 7 матчей, 4 отменили 4 победы Роберта и еще 3 для выигрыша пиццы.

Загадка: Я трехзначное число. Моя вторая цифра в 4 раза больше третьей. Моя первая цифра на 3 меньше, чем вторая. Кто я?
Ответ: 141

Загадка: Я складываю пять к девяти и получаю два.Ответ правильный, но как?
Ответ: Когда будет 9 часов утра, прибавьте к нему 5 часов, и вы получите 14 часов.

Загадка: В зоопарке 100 пар собак; На каждую собаку рождаются пары младенцев. К сожалению, 23 собаки не выжили. Сколько всего собак останется?
Ответ: 977 собак
Пояснение: 100 x 2 = 200; 200 +800 = 1000; 1000-23 = 977

Загадка: Группа студентов стояла под палящим солнцем лицом к западу во время марша, прошедшего мимо.Вождь крикнул им: Поверните направо! О очередь! Левый поворот! В конце этих команд, в каком направлении сейчас смотрят ученики?
Ответ: Восток. (Пояснение: они повернутся на 90 градусов при повороте вправо, и они повернутся на 180 градусов при повороте, и, наконец, они повернутся на 90 градусов при повороте влево. Таким образом, ученики теперь смотрят на восток.) ​​

Загадка: Половина – это треть. Что это?
Ответ: 1 1/2.

Загадка: Во время доставки Том может поместить в картонную коробку 10 маленьких или 8 больших коробок.Всего за одну партию было отправлено 96 коробок. Маленьких ящиков было меньше, чем больших. Какое общее количество коробок он отправил?
Ответ: 11 коробок ( Пояснение: 4 маленьких коробки (4 10 = 40 коробок) + 7 больших коробок (7 8 = 56 коробок). Итак, 96 коробок и 11 коробок всего)

Загадка: Когда Мигелю было 6 лет, его младшей сестре Лейле было полгода. Если Мигелю сегодня 40 лет, сколько лет Лейле?
Ответ: Ей 37 лет.

Загадка: Вам дано 3 положительных числа. Вы можете сложить эти числа и умножить их вместе. Результат будет таким же. Какие числа?
Ответ: 1, 2 и 3

Загадка: Если полторы курицы откладывают полтора яйца за полтора дня, сколько яиц откладывает полдюжины куриц за полдюжины дней? Ответ: 2 дюжины.

Загадка: Что можно поставить между 7 и 8, чтобы результат был больше семи, но меньше восьмерки?
Ответ: Десятичный.7,8 больше 7, но меньше 8

Загадка: Тома попросили нарисовать числа за пределами 100 квартир, что означает, что ему придется раскрасить числа от 1 до 100. Можете ли вы подсчитать, сколько раз ему придется раскрасить число 8?
Ответ: 20 раз (8, 18, 28, 38, 48, 58, 68, 78, 80, 81, 82, 83, 84, 85, 86, 87, 88, 89, 98)

Загадка: Какое максимальное количество раз вы можете вычесть число 5 из 25?
Ответ: Только один раз.Это потому, что когда вы вычитаете 5 в первый раз, получается число 20, затем 15 и так далее.

Загадка: Что весит больше 16 унций соды или фунта чистого золота?
Ответ: Ни то, ни другое. Оба они весят одинаково!

Загадка: Леон работает в аквариуме. Когда он пытается поместить каждую черепаху в отдельный резервуар, у него на одну черепаху слишком много. Но если он ставит две черепахи на танк, у него слишком много на танке. Сколько черепах и сколько танков у Леона?
Ответ: У него 3 танка и 4 черепахи.

Загадка: Общая стоимость пары туфель и толстовки составляет 150 долларов. Толстовка с капюшоном стоит на 100 долларов больше, чем пара обуви. Сколько стоит каждый предмет?
Ответ: Толстовка с капюшоном стоит 125 долларов, обувь – 25 долларов

Загадка: У вас есть две монеты США общей стоимостью 0,30 доллара. Один из них не никель. Какие две монеты?
Ответ: Один – четверть, а другой – никель.

Загадка: Яйца стоят 0 долларов.12 дюжина. Сколько яиц можно получить за доллар?
Ответ: 100 яиц по пенни каждое.

Загадка: Утка – 9 долларов, паук – 36 долларов, пчела – 27 долларов. На основании этой информации, сколько денег будет отдано кошке?
Ответ: 18 долларов (4,50 доллара за ногу)

Загадка: «Сколько стоит этот мешок картошки?» спросил мужчина. «32 фунта, разделенные на половину собственного веса», – сказал бакалейщик. Сколько весила картошка?
Ответ: 8 фунтов.

Загадка: Я трехзначное число. Моя цифра десятков на шесть больше, чем моя цифра единиц. Моя цифра сотен на восемь меньше, чем моя цифра десятков. Какой я номер?
Ответ: 193

Загадка: Мужчина вдвое старше своей младшей сестры. Он тоже вдвое младше своего отца. Через 50 лет возраст его сестры станет вдвое меньше возраста их отца. Какого возраста сейчас мужчина?
Ответ: 50 лет.

Загадка: Как сложить восемь четверок, чтобы в сумме получилось 500?
Ответ: 444 + 44 + 4 + 4 + 4 = 500

Загадка: Если семь человек встретятся друг с другом и пожмут друг другу руки только один раз, сколько рукопожатий будет?
Ответ: 21

Загадка: Если четыре человека могут построить четыре стола за четыре часа, сколько столов восемь человек могут построить за восемь часов?
Ответ : 16 таблиц

Загадка: Когда Лизе было 6 лет, ее сестра Люси была вдвое моложе.Если Люси сегодня 40 лет, сколько лет Люси?
Ответ: 37
Пояснение: Люси на 3 года младше Лизы.

Загадка: Если вы покупаете петуха для откладывания яиц и рассчитываете получать по три яйца каждый день на завтрак, сколько яиц у вас будет через три недели?
Ответ: Нет. Петухи яиц не откладывают.

Загадка: У фермера на земле 19 овец. Однажды обрушивается сильный шторм, и все, кроме семи, разбегаются.Сколько овец осталось у фермера?
Ответ: Семь. Все, кроме семи, сбежали.

10 логических вопросов на собеседовании, которые могут быть заданы на собеседовании (с ответами)

Перед следующим собеседованием вы должны быть готовы ответить на логические вопросы. В этой статье мы рассмотрим некоторые общие логические вопросы, которые задают интервьюеры, и способы на них ответить.

Подробнее: Как подготовиться к интервью

Какие вопросы на собеседовании логичны?

Логические вопросы интервью включают решение головоломок или загадок какого-либо типа, чтобы показать интервьюеру ваши навыки критического мышления, навыки решения проблем и аналитические способности.Логические вопросы для целей собеседования можно использовать для оценки этих навыков, а также для определения того, как вы запрашиваете информацию, используете ресурсы и работаете под давлением.

Хотя логические вопросы иногда могут быть трудными, вы можете повысить свои шансы на успешные ответы на эти типы вопросов собеседования, подготовившись заранее. Есть три основные категории логических вопросов интервью:

Загадки

Логические вопросы в форме загадок часто встречаются во время собеседований, посвященных головоломкам.Такие вопросы представляют собой загадку, на которую вы должны ответить. В таких логических вопросах правильный ответ часто не является самым важным аспектом. Интервьюер задает эти типы вопросов, чтобы оценить вашу способность творчески мыслить, решать проблемы с помощью уникальных подходов и использовать ваши навыки критического мышления для поиска способов поиска решений.

Проблемы с счислением

Проблемы с счислением также могут быть типом вопросов логического собеседования, с которыми вы можете столкнуться.Эти вопросы могут потребовать использования математических вычислений, таких как определение вероятности или использование дедуктивных и индуктивных рассуждений для решения проблемы. Интервьюер ставит задачи со счетом, чтобы лучше оценить ваши аналитические навыки и навыки решения проблем. Опять же, процесс, который вы используете для поиска решения, может быть более важным, чем получение правильного ответа.

Поиск сумм

Как и в задачах с математикой, поиск сумм для решения задач-головоломок может потребовать навыков математического и критического мышления.Вопросы, требующие определения сумм, могут показать интервьюеру, как вы рассуждаете и планируете процесс решения проблем. Использование вопросов о сумме также может помочь интервьюеру лучше понять, как вы подходите к незнакомым сценариям и решаете проблемы. Как правило, на эти вопросы может быть конкретный ответ, и для некоторых интервьюеров поиск правильного ответа может быть столь же важным, как и то, как вы к нему пришли.

Связанные: 12 сложных вопросов и ответов на собеседовании

Логические вопросы интервью и примеры ответов

Ниже приведены примеры ответов на 10 вопросов собеседования, которые создают логическую проблему.

Три лампочки

В этом логическом вопросе вы стоите в комнате с тремя выключателями. Все переключатели соответствуют трем различным лампочкам в соседней комнате, в которую вы не можете видеть. Когда все переключатели света начинаются в выключенном положении, как узнать, какой переключатель к какой лампочке подключается?

Ответ: «Я бы включил первый переключатель и оставил его включенным в течение нескольких минут. Затем я бы выключил первый переключатель и быстро включил второй переключатель.Затем, проверяя комнату, я видел, что вторая лампочка включается, и щупал две другие лампочки, чтобы увидеть, какая из них теплее. Более теплая лампочка – это та, которую я только что выключил, поэтому она принадлежит первому переключателю, а горящая лампочка – второму переключателю. Третий переключатель будет принадлежать лампочке, которая выключена и самая холодная на ощупь ».

Крышки люков

Этот логичный вопрос можно представить как загадку, и существует несколько вариантов формата этой головоломки.Вот вопрос:

Почему крышки люков круглые?

Пример ответа: “Крышки люков круглые, поэтому их никогда не нужно поворачивать, чтобы снова надеть крышку. Кроме того, круги – единственная геометрическая форма, которая не упадет на себя в отверстии аналогичной формы. , поэтому она никогда не должна упасть. Например, квадратная крышка люка может упасть в квадратное отверстие, если вставить ее по диагонали “.

Фермер, лисица и цыпленок

В этой загадке вы сталкиваетесь с проблемой фермера: переправить своих животных и урожай через реку.Вот вопрос:

Фермеру нужно перейти реку со своей лисой, курицей и мешком кукурузы. Однако лодка может вместить только фермера и еще одну вещь за раз. Проблема в том, что лиса и курица оба голодны, поэтому, если он оставит лису и курицу вместе, лиса может съесть курицу. Если он оставит курицу и кукурузу вместе, курица может съесть кукурузу.

Так как же фермеру безопасно переправить всех через реку?

Пример ответа: « Сначала фермер должен перевезти курицу.Затем он может вернуться за лисой. Когда он придет на другую сторону, чтобы высадить лису, он может забрать курицу с собой, чтобы забрать кукурузу. Фермер бросает курицу и переправляет кукурузу на другой берег реки. Наконец, фермер возвращается за цыпленком и продолжает свой путь на другой берег реки, ничего не съев ».

Четыре имени

Этот вопрос – довольно простая загадка, и на самом деле она зависит только от ваших навыков активного слушания.Вот вопрос:

У матери Джейн четверо детей. Первого ребенка зовут Весна, второго ребенка зовут Лето, а третьего ребенка зовут Осень. Как зовут четвертого ребенка?

Пример ответа: «Четвертого ребенка зовут Джейн, потому что ее назвали в самом начале».

Бросок кубиков

Эта головоломка для математических навыков может принимать различные формы, однако основа обычно одна и та же:

Если я брошу два кубика, какова вероятность того, что я получу два при выпадении второй рулон?

Пример ответа: «Вероятность выпадения двойки на шестигранной кости составляет 1/6.Для двух кубиков у меня по-прежнему был бы один шанс из шести бросить по два для каждого, поэтому я бы умножил обе вероятности, чтобы получить окончательную вероятность выпадения двух двоек: 1/6 x 1/6 = 1/36. Таким образом, вероятность выпадения двух двоек составляет один к 36 ».

Заполнение кувшинов

Этот вопрос может потребовать некоторых математических вычислений:

Если у вас есть трехгаллонный кувшин и пятигаллонный кувшин , как вы можете отмерить ровно четыре галлона воды?

Пример ответа: «Сначала я бы наполнил трехгаллонный кувшин и налил его в пятигаллонный кувшин.Затем я снова наполнял трехгаллонный кувшин и полностью заполнял пятигаллонный кувшин. Поскольку пятигаллонный кувшин уже вмещает три галлона, в трехгаллонном кувшине останется один галлон воды. Затем я опорожнял полный пятигаллонный кувшин и выливал в него один галлон воды, который я сэкономил. Наконец, я наполняю трехгаллонный кувшин и добавляю его к галлону воды в пятигаллонном кувшине, в результате чего получается четыре галлона ».

Прогнозирование дождя

Этот логичный вопрос интервью может также полагайтесь на некоторые базовые вычисления вероятности:

Допустим, вы едете в Вашингтон Д.C. в гости к друзьям. Вы звоните трем своим друзьям, чтобы узнать, идет ли в городе дождь. Какова вероятность того, что в Вашингтоне действительно идет дождь?

Пример ответа: «Поскольку мне нужен только один друг, чтобы рассказать правду о том, что идет дождь, было бы проще рассчитать вероятность того, что все три друга лгут. Таким образом, я могу подсчитать, что у каждого человека есть 1/3 шанса солгать о дожде. Если умножить, я получу 1/27 шанса, что все трое друзей солгали о погоде.Это означает, что вероятность того, что на самом деле идет дождь, составляет 26/27. Это равняется 96% вероятности того, что когда я приеду в город, пойдет дождь ».

Шаров для пинг-понга в самолете

Этот вопрос может основываться на некоторой фактической информации, однако некоторых интервьюеров может больше беспокоить то, как вы обрабатывать информацию, использовать ресурсы и создавать решение.Кроме того, с подобными вопросами вы можете попросить дополнительную информацию, которая поможет вам в расчетах.Вот вопрос:

Сколько мячей для пинг-понга поместится в Боинг 747?

Пример ответа: «Во-первых, мне нужно знать объем мяча для пинг-понга, а также объем внутри Boeing 747. Зная эту информацию, я могу затем разделить объем самолета на объем. одного шара для пинг-понга. В результате получится приблизительное количество мячей для пинг-понга, которое поместится внутри самолета “.

Восхождение на стену

В этом вопросе по математике основное внимание уделяется шаблонам чисел и способам обработки информации для расчета результата.Вот вопрос:

Представьте, что вы стоите у подножия 30-футовой стены. Вы начинаете лазать, и каждый час вы поднимаетесь на три фута, но скользите вниз на два фута. Предполагая, что вы будете придерживаться этой схемы лазания, сколько времени вам понадобится, чтобы подняться на вершину стены?

Пример ответа: «Во-первых, чтобы найти общую сумму, которую я поднял за час, я убираю два фута в час, когда я соскальзываю. Это дает мне в общей сложности один фут за час подъема, что результаты через 27 часов.Если предположить, что я не упаду в следующий час, мне понадобится еще три фута, чтобы достичь вершины на высоте 30 футов. Однако на каждые три фута, которые я набираю, я падаю, поэтому вычитание двух футов за скольжение в течение последнего часа дает в общей сложности 28 часов, потраченных на лазание ».

Укладка пенсов

интервьюера могут больше заботить ваши навыки решения проблем, чем ответ, к которому вы пришли. Тем не менее, вы можете использовать основные математические расчеты и информацию, полученную от интервьюера, чтобы ответить на этот вопрос:

Сколько пенни, если они сложены поверх друг друга, потребуется ли, чтобы добраться до вершины Эмпайр-стейт-билдинг?

Пример ответа: «Во-первых, мне нужно знать, какой высоты Эмпайр-стейт-билдинг, а также сколько пенни, когда он укладка плоская.Предполагая, что один пенни составляет 1/4 дюйма в высоту, я могу разделить высоту здания на высоту пенни, чтобы получить количество пенни, которое мне нужно будет сложить ».

Подробнее: 125 Common Вопросы и ответы на собеседовании (с советами)

Советы по ответам на логические вопросы на собеседовании

Практика того, как вы будете отвечать на некоторые из этих логических вопросов собеседования, определенно поможет подготовиться к собеседованию. Вы также можете воспользоваться следующими советами, которые помогут вам добиться успеха на собеседовании:

Просите пояснений, если она вам нужна

Иногда логические вопросы собеседования могут основываться на знании некоторой фактической информации, такой как суммы или определенные измерения.Вы можете спросить эту информацию у интервьюера или использовать предполагаемые значения.

Покажите, как вы решаете проблему

Вы также можете использовать лист бумаги и карандаш, чтобы помочь вам решить логические вопросы собеседования, особенно вопросы, связанные с математикой и определением сумм. Это также может позволить интервьюеру увидеть ваш процесс решения проблемы.

Дайте ответ

Даже если вы пришли к неправильному ответу, постарайтесь всегда предложить решение.Скорее всего, интервьюер оценивает актуальность вашего подхода и методов решения проблем, и ваш ответ может быть не так важен для него, как ваша способность творчески мыслить и пытаться решать проблемы.

Рассказ загадок и ответ | Лучшие загадки и головоломки

Жил-был кондуктор автобуса, который очень грубо обращался со всеми пассажирами. В один прекрасный день в автобус попыталась сесть молодая очаровательная женщина. Но автобус не останавливал. В результате девушка попала под автобус и погибла.Взволнованные пассажиры отвезли кондуктора в отделение милиции. Полиция доставила его в суд. В суде судья вынес ему смертный приговор.

Затем его помещают в камеру поражения электрическим током. В центре комнаты стоит стул, а сразу за ним лежит кожура банана. Проводник привязан к этому стулу и подвергается воздействию высокого напряжения. Но, ко всеобщему удивлению, он способен пережить течение, которого достаточно, чтобы убить сразу двух человек.Судья решает, что это чудо, и освобождает кондуктора.

Затем дирижер возвращается к своей профессии. Через несколько месяцев женщина средних лет пытается сесть в автобус. Кондуктор не останавливает автобус, в результате женщина попадает под автобус, который убивает ее на месте. Взволнованные пассажиры отвозят его в полицию. Полиция доставит его в суд для судебного разбирательства. В суде судья выносит решение о смертной казни дирижеру.

Проводника снова переносят в камеру поражения электрическим током.В этой камере посреди комнаты стоит стул, а за ним лежит кожура банана. Он привязан к стулу и подвергается воздействию высокого напряжения. К всеобщему удивлению, он снова выжил. Таким образом, судья освобождает его.

Дирижер возвращается к своей профессии. Через несколько месяцев. Пожилая женщина пытается сесть в автобус. Кондуктор, вспомнив свои предыдущие действия, останавливает автобус. Но, к сожалению, старушка поскользнулась, взбираясь на автобус, и скончалась от полученных травм.Пассажиры снова отвозят его в полицию. Полиция возвращает его в суд. Хотя судья знает, что он не виновен, он приговаривает его к смертной казни, зная его предыдущие записи.

Проводника снова переносят в камеру поражения электрическим током. Он привязан к стулу в центре комнаты, за которым находится банановая кожура. Он попадает под ток, и на этот раз его мгновенно убивают.

Почему он умер в этот раз, а не в предыдущий?

На этот раз он умер, потому что был «хорошим дирижером».Хороший проводник эффективно пропускает электрический ток. Хотя в обоих предыдущих случаях он был «плохим проводником», поэтому ток не мог пройти через него.

Топ-15 логических головоломок и головоломок: советы и рекомендации по программированию на Java.

Ведущие инвестиционные банки или известные компании задавали кандидатам множество головоломок по программированию на Java, а также вопросы-головоломки. Эти логические головоломки в java-интервью используются, чтобы проверить, может ли кандидат мыслить нестандартно, сталкиваясь со сложными и сложными проблемами.Давайте обсудим советы, как эффективно решать подобные вопросы.

Советы по решению головоломок и головоломок программирования на Java

1. Отвечая на головоломки программирования на Java, помните, что очевидный ответ почти никогда не является правильным.

2. Если решение, которое кажется логичным, неверно, вы сделали ложное предположение. Разделите свои предположения на категории и попытайтесь определить те, которые ложны.

3. Не пугайтесь сложности. Попробуйте подзадачу, упрощенную версию или несколько примеров.Будьте терпеливы, продолжайте работать и продолжайте говорить.

4. Когда вы застряли на простой ограниченной проблеме, подумайте обо всех возможностях, чтобы определить ту, которую вы упускаете.

Пазлы и вопросы для программирования на Java

Q1 Вы стоите в коридоре рядом с тремя выключателями света, все из которых выключены. Каждый выключатель управляет отдельной лампой накаливания в комнате в конце коридора. Вы не можете видеть свет от переключателей.Определите, какой свет соответствует каждому переключателю. В комнату с освещением можно зайти только один раз.

Q2 Как разрезать прямоугольный торт на две равные части когда кто-то уже снял с него прямоугольный кусок? В удаленный кусок может быть любого размера и ориентации. Вам разрешено просто один прямой срез.

Q3 У вас есть восемь шариков и весы с двумя панелями. Все шарики весят одинаково, за исключением одного, который тяжелее всех остальных.В остальном мрамор невозможно отличить. Вы не можете делать никаких предположений о том, насколько тяжелее тяжелый мрамор. Какое минимальное количество взвешиваний необходимо для уверенности в идентификации тяжелого мрамора? Теперь обобщим вопрос. Какое минимальное число взвешиваний позволяет найти тяжелый шарик среди n шариков?

Q4 Между Кристиной и Сири 21 доллар. У Кристины на 20 долларов больше, чем у Siri. Сколько у каждого. Вы не можете использовать дроби в ответе

Q5 Группа из четырех путешественников ночью подходит к шаткому мосту.Мост может выдержать вес не более двух путешественников. время, и его нельзя пересечь без фонарика. Путешественники есть среди них один фонарик. Каждый путешественник идет по разному скорость: первый может пересечь мост за 1 минуту, второй – за 2 минут, третий – через 5 минут, а четвертый – за 10 минут до переходить мост. Если двое путешественников переходят дорогу вместе, они идут по скорость более медленного путешественника. Какое наименьшее количество времени все путешественники могут перейти с одной стороны моста на другую?


Q6 У вас есть 10 банок, наполненных шариками.Каждый шарик весит 10 граммов, за исключением одной банки с дефектными шариками, которая весит 9 граммов. Учитывая весы для взвешивания, как найти банку с дефектными шариками всего за одно измерение. Здесь весы – это современные электронные весы для измерения веса.

Q7 Человек стреляет в жену. Затем держит ее под водой 5 минут. Наконец он ее вешает. Но через 10 минут они оба вместе выходят и вместе наслаждаются чудесным ужином. Как это может быть?

Q8 Отец и сын едут на машине и внезапно там машина попала в аварию.Отец скончался на месте, но ребенок срочно доставлен в больницу. Когда он попадает в больницу, врач говорит: «Я не могу оперировать этого ребенка, он мой сын!» Как это может быть?

Q9 Три муравья находятся в разных вершинах треугольник. Какова вероятность столкновения (между любыми двумя или всеми из них) если они начнут ходить по сторонам треугольника? сходным образом найти вероятность столкновения с n муравьями на вершине n многоугольник.

Q10 Начиная с одного конца, одно полное прожиг (горение с одного конец к другому концу) требуется по 1 часу на две палочки.При условии, что палочки не горят с постоянной скоростью, т.е. некоторые части могут гореть быстрее, чем другая часть палки. Вам нужно отмерить 45 минут с помощью эти две палочки и зажигалка.

Более сложные вопросы

Q11 В небольшой школе всего со 100 шкафчиками в последний день учебы проводится следующий ритуал: ученик входит в холл и встает у своих закрытых шкафчиков. По первому свистку ученики открывают все шкафчики. По второму свистку ученики закрывают каждый второй шкафчик (шкафчики 2, 4, 6 и т. Д.).захлопываются). По третьему свистку ученики переключают каждый третий шкафчик. «Переключение» означает закрытие, если оно открыто, и открытие, если оно закрыто. Они переключают шкафчики 3,6,9 и т. Д. На 99. По четвертому свистку они переключают каждый четвертый шкафчик. По пятому свистку переключают каждый пятый шкафчик и так далее.
По свистку сотки ученик, стоящий рядом со шкафчиком 100 (и только этот ученик), переключает свой шкафчик. Сколько шкафчиков тогда открыто.

Q12 Каждый мужчина в деревне, состоящей из пятидесяти пар, изменил своей жене.Каждая женщина в деревне мгновенно узнает, когда другой мужчина, кроме ее мужа, занимается благотворительностью (имеется в виду мужчина, который легко вступает в случайные сексуальные отношения с женщинами), но не тогда, когда ее собственный муж («всегда узнает последним»). Закон о терпимости к супружеской неверности требует, чтобы женщина, которая может доказать, что ее муж неверен, должна убить его в тот же день. Ни одна женщина не будет думать о нарушении этого закона. неправильно), посещает деревню.Она объявляет, что по крайней мере один муж изменил ей. Что происходит ?

Q13 Пять пиратов на острове должны разделить между собой сотню золотых монет. Они делят добычу следующим образом: старший пират предлагает разделение, и все голосуют за него. Если хотя бы половина пиратов проголосует за предложение, они делят монеты таким же образом, если нет, они убивают старшего пирата и начинают все сначала. Самый старший (выживший) пират предлагает свой план разделения, и они голосуют по тем же правилам и либо делят добычу, либо убивают старшего пирата, в зависимости от обстоятельств.Процесс продолжается до тех пор, пока не будет принят один план. Предположим, вы старший пират. Какую дивизию вы предлагаете? (Пираты все чрезвычайно логичны и жадны, и все хотят жить).

Q14 Вы находитесь в лодке в самом центре идеально круглого озера. На берегу озера живет гоблин. Гоблин хочет сделать вам плохие вещи. Гоблин не умеет плавать и у него нет лодки. При условии, что вы можете добраться до берега – а гоблина нет рядом и ждет, чтобы схватить вас – вы всегда можете обогнать его на суше и уйти.
Проблема в следующем: гоблин может бежать в четыре раза быстрее, чем максимальная скорость вашей лодки. У него прекрасное зрение, он никогда не спит и очень логичен. Он сделает все, что в его силах, чтобы поймать вас. Как бы вы сбежали от гоблина?

Q15 Один из ваших сотрудников настаивает на ежедневной оплате золотом. У вас есть золотой слиток, стоимость которого равна семидневной зарплате этого сотрудника. Планка уже разделена на семь равных частей. Если вам разрешено делать только два сокращения в планке и вы должны рассчитываться с сотрудником в конце каждого дня, как вы это делаете?

# Weird Logical Thinking Question

1.Сколько заправок в США (вашей стране)?

2. Как бы вы нашли конкретную книгу в большой библиотеке? Нет системы каталогизации и библиотекаря, который мог бы вам помочь?

В ближайшее время поделимся ответами, а пока попробуем решить вопросы самостоятельно. Если решите какие-то вопросы, то укажите ответы в комментарии.

советов и приемов для решения вопросов на основе головоломок

Что такое логические головоломки?

Головоломки на логическое рассуждение требуют, чтобы вы проанализировали данную информацию, выбрали важную информацию и опускали информацию, которая не требуется при решении данного набора вопросов.Головоломки на рассуждение в основном основаны на кровном родстве, договоренностях, неравенстве и т. Д. Иногда вопросы проверяют ваше нестандартное мышление и могут не требовать никаких дедукций или анализа.

Основные шаги для решения логических головоломок
  1. Взгляните на вопрос.
  2. Разработайте общее представление о теме проблемы.
  3. Выберите данные, которые дают вам некоторую конкретную информацию из общей предоставленной информации.Кроме того, выберите данные, которые помогают исключить определенные возможности.
БЕСПЛАТНЫЕ живые мастер-классы от нашего звездного факультета с более чем 20-летним опытом. Зарегистрируйтесь сейчас
  1. Полезная вторичная информация: Как правило, первые несколько предложений приведенных данных дают вам основную информацию, необходимую для понимания общей идеи проблемы. Это называется «полезной вторичной информацией».
  2. Актуальная информация: Данные, оставшиеся после откладывания полезной вторичной информации, называются фактической информацией.Пытаясь решить проблему, вы должны начать с фактической информации, в то время как полезную вторичную информацию следует иметь в виду.
  3. Отрицательная информация: Некоторые части фактической информации могут содержать отрицательную информацию. Отрицательная информация ничего нам не сообщает, но дает возможность исключить возможность. Такие предложения, как «P не является матерью Q» или «B не является горной станцией», называются отрицательной информацией.
Что нужно помнить при решении логических головоломок:
  1. Во-первых, соберите всю прямую информацию.
  2. Расположите прямую информацию в табличном формате.
  3. Затем определите всю негативную информацию и отметьте ее в своей таблице в соответствии с приведенными данными.
  4. Остальные факты можно узнать из косвенной информации.
  5. Наиболее важным моментом является то, что при некоторых условиях / ах будет более одной возможности.
  6. Итак, по возможности разложите их по разным таблицам.
  7. Отклонить все таблицы, которые нарушают другую информацию, указанную в головоломке.
  8. Следуя этому процессу, вы получите правильный ответ.
  9. Большинство головоломок непростые. Итак, позаботьтесь о каждом слове и предложении.
Решенный пример

Пример: Внимательно прочтите данную информацию и ответьте на следующие вопросы:

  1. Пятеро друзей Ами, Бими, Кэл, Дэн и Эко путешествовали из Дарджилинга в пять разных стран США, Великобритании, Китая, Канады и России на 5 различных видах транспорта: велосипед, ракета, машина времени, яхта и велосипед.
  2. Тот, кто путешествовал по Китаю, путешествовал не на велосипеде.
  3. Кэл отправился в Канаду на яхте, а Бими отправился в Великобританию на машине времени.
  4. Дэн путешествовал на Байке, а Эко путешествовал на Ракете.
  5. Дарджилинг не связан Cycle с США и Китаем.

1. Какая из следующих комбинаций человека и режима не исправлена?

А. Ами – Цикл

Б. Бими – Машина времени

C. Cal – Яхта

Д.Дэн – Велосипед

E. Eco – Машина времени

Решение: (A) Вид транспорта: Кэл путешествует на яхте, Бими – на машине времени, Дэн – на велосипеде, а Эко – на ракете. Теперь осталась только Ами. Итак, Ами путешествует по Циклу.
(B) Место путешествия: Бими в Великобританию, Кэл уезжает в Канаду. Теперь велосипедный транспорт недоступен для США и Китая. Таким образом, Ами, путешествующая на Cycle, отправляется в Россию. Дэн путешествует на велосипеде и, следовательно, согласно (ii), не поехал в Китай. Итак, Дэн едет в США. Теперь осталась только Эко.Итак, Эко уезжает в Китай. Ясно, что неправильная комбинация – Eco – Time Machine. Итак, ответ – вариант E.

2. Какая из следующих комбинаций верна для Дэна?

A. Китай – цикл

B. США – цикл

C. США – велосипед

D. Недостаточные данные

E. Ни один из этих

Решение: (A) Вид транспорта: Кэл путешествует на яхте, Бими на машине времени, Дэн на велосипеде и Эко на ракете. Теперь осталась только Ами.Итак, Ами путешествует по Циклу.

Б. Место путешествия: Бими в Великобританию, Кэл отправляется в Канаду. Теперь велосипедный транспорт недоступен для США и Китая. Таким образом, Ами, путешествующая на Cycle, отправляется в Россию. Дэн путешествует на велосипеде и, следовательно, согласно (ii), не поехал в Китай. Итак, Дэн едет в США. Теперь осталась только Эко. Итак, Эко уезжает в Китай.

Ясно, что правильная комбинация для Дэна – US – Bike. Итак, ответ – вариант C.

3. Какая из следующих комбинаций места и режима неправильная?

А.Китай – цикл

B. Великобритания – Машина времени

C. Канада – Яхта

D. США – велосипед

E. Россия – цикл

Решение: (A) Вид транспорта: Кэл путешествует на яхте, Бими на машине времени, Дэн на велосипеде и Эко на ракете. Теперь осталась только Ами. Итак, Ами путешествует по Циклу.

(B) Место путешествия: Бими в Великобританию, Кэл уезжает в Канаду. Теперь велосипедный транспорт недоступен для США и Китая. Таким образом, Ами, путешествующая на Cycle, отправляется в Россию.Дэн путешествует на велосипеде и, следовательно, согласно (ii), не поехал в Китай. Итак, Дэн едет в США. Теперь осталась только Эко. Итак, Eco едет в Китай

Ясно, что неправильная комбинация – Китай — Цикл. Итак, ответ – вариант А.

4. Какой из следующих режимов использовал человек, путешествующий в Китай?

A. Цикл

B. Ракета

C. Машина времени

D. Яхта

E. Велосипед

Решение: (A) Вид транспорта: Кэл путешествует на яхте, Бими на машине времени, Дэн на велосипеде и Эко на ракете.Теперь осталась только Ами. Итак, Ами путешествует по Циклу.

(B) Место путешествия: Бими в Великобританию, Кэл уезжает в Канаду. Теперь велосипедный транспорт недоступен для США и Китая. Таким образом, Ами, путешествующая на Cycle, отправляется в Россию. Дэн путешествует на велосипеде и, следовательно, согласно (ii), не поехал в Китай. Итак, Дэн едет в США. Теперь осталась только Эко. Итак, Eco едет в Китай

Очевидно, Эко отправился в Китай на Ракете. Итак, ответ – вариант Б.

5. Кто из следующих побывал в Китае?

А.Cal

Б. Дан

К. Эко

D. Ни один из этих

E. Недостаточные данные

Решение: (A) Вид транспорта: Кэл путешествует на яхте, Бими на машине времени, Дэн на велосипеде и Эко на ракете. Теперь осталась только Ами. Итак, Ами путешествует по Циклу.

(B) Место путешествия: Бими в Великобританию, Кэл уезжает в Канаду. Теперь велосипедный транспорт недоступен для США и Китая. Таким образом, Ами, путешествующая на Cycle, отправляется в Россию. Дэн путешествует на велосипеде и, следовательно, согласно (ii), не поехал в Китай.Итак, Дэн едет в США. Теперь осталась только Эко. Итак, Эко уезжает в Китай.

Очевидно, Эко поехал в Китай. Итак, ответ – вариант C.

Получите доступ к тщательно подобранным электронным книгам академических экспертов для успешной сдачи конкурсных экзаменов.

Логические головоломки: концепция и практика

40 сложных загадок, которые оставят вас в тупике

Когда вы в последний раз загадывали загадку? Загадки – это интересный способ проверить свои навыки решения проблем, побуждая вас исследовать свои предположения, искать альтернативные значения и мыслить более творчески.Ценность хорошей загадки для мозга не исчезает по мере взросления; подобные словесные головоломки помогут вам оставаться в уме. Так что читайте 40 сложных загадок, которые заставят вас задуматься.

1. Приближается зима

Загадка: Я принадлежу декабрю, но не другому месяцу. Я не праздник. Что я?

Это не имеет отношения к дню или температуре…

Ответ: Буква «д»

2. Старинный вопрос

Загадка: Когда Билли спрашивают, сколько ему лет, он отвечает: «Через два года я буду вдвое старше, чем был пять лет назад.«Сколько ему лет?

Просто посчитайте.

Ответ: 12 лет

3. Кофеин высокий

Загадка: Джордж, Хелен и Стив пьют кофе. Берт, Карен и Дэйв пьют «Пепси». Следуя этой схеме, Элизабет пьет кофе или газировку?

Обратите внимание на гласные.

Ответ: Кофе, как и все остальные названия с двумя буквами е. (Те, у кого была буква «е», пили пепси.)

4. Нечетное слово

Загадка: Одно из этих слов не принадлежит: Что это такое и почему? Драка, Морковь, Смена, Клевер, Правильный, Священный, Камень, Семьдесят, Качели, Путешествие.

Это не имеет отношения к порядку, в котором они находятся в

Ответ: Морковь. Это единственное слово, которое не превращается в другое слово, когда вы удаляете первую и последнюю буквы.

5. Седло вверх

Загадка: Без уздечки и седла, я катаюсь верхом. А те, на которых я езжу, благодаря моей почти слепоте, стали видеть. Что я?

Присмотритесь, и ответ должен появиться в фокусе

Ответ: Очки для глаз

6.Плавающий на ветру

Загадка: Белая птица без перьев. Полет из рая. Полет над морем и сушей. Умираю в моей руке. Что это?

Думаешь, ты понял? Становится холоднее…

Ответ: Снежинка

7. Горячий отпуск

Загадка: Мой гром предшествует молнии. Моя молния опережает облака. Мой дождь сушит всю землю, к которой прикасается. Что я?

Мы могли бы также добавить, что вы не хотите быть рядом со мной, когда я взорву свою вершину.

Ответ: Вулкан

8. Странное тело

Загадка: У кого есть шея и нет головы, две руки и нет рук?

Может выглядеть забавно или морщинисто.

Ответ: Рубашка

9. Думайте меньше

Загадка: Я везде и часть каждого. Я нахожусь в конце пространства, времени и самого существования. Что я?

Мыслите скорее буквально, чем метафизически.

Ответ: Буква «е»

10.Напряженный и шумный день

Загадка: Кто днем ​​сидит у окна, идет к столу обедать, а ночью прячется?

Он также не особо желанный гость на обеде.

Ответ: Муха

11. Личное дело

Загадка : Это принадлежит вам, но все остальные используют его.

Не спотыкайтесь, думая, что это реальный объект…

Ответ : Ваше имя

12. Деревянная доставка

Загадка : Я выхожу из шахты и окружен лесом.Я помогаю другим выразить себя. Что я?

Благодаря текстовым сообщениям и электронной почте я не привык так сильно, как раньше.

Ответ : Грифель

13. Распространенная ошибка

Загадка : Какое слово в словаре всегда пишется неправильно?

Ответ прямо в вопросе.

Ответ : «Неправильно»

14. Где в мире

Загадка : У меня есть города, но нет домов.У меня есть горы, но нет деревьев. У меня есть берега, но нет песка. Что я?

Подумайте о «большой картине» с этим.

Ответ : Карта

15. Прочтите все об этом

Загадка : Какое английское слово состоит из трех последовательных двойных букв?

Это слово на самом деле относится к тому, кто любит числа.

Ответ : Бухгалтер

16. Трудно увидеть

Загадка : Вы находитесь в темной комнате со свечой, дровяной печью и газовой лампой.У вас только одна спичка, так что вы зажигаете в первую очередь?

Это очевидно, когда знаешь, что искать.

Ответ : совпадение

17. Просветительный вопрос

Загадка : Вы измеряете мою жизнь часами, а я служу вам истечением срока. Я быстрая, когда я худой, и медленная, когда толстая. Ветер – мой враг.

«Быстрый» не означает, что он движется к вам.

Ответ : Свеча

18.Отплыть

Загадка : Вы видите лодку, полную людей. Он не затонул, но когда вы посмотрите еще раз, вы не увидите ни одного человека на лодке. Почему?

Сосредоточьтесь на слове «одинокий», чтобы найти ответ на этот.

Ответ : Все пассажиры женаты.

19. Не знаю, что у тебя есть, пока оно не исчезнет

Загадка : Что становится больше, чем больше вы забираете?

Удачи вам выбраться из этого.

Ответ : дыра

20. Иметь и держать

Загадка : Что можно держать правой рукой, но никогда – левой?

Ответ прямо перед вами.

Ответ: Левая рука

21. Ключевой вопрос

Загадка : У меня есть ключи, но нет замков. У меня есть место, но нет места. Вы можете войти, но не можете выйти на улицу. Что я?

Если вы похожи на большинство современных рабочих, вы обычно тратите на это много времени.

Ответ : клавиатура компьютера

22. Поношенный

Загадка : Это пальто лучше всего класть на мокрый.

Это легко, если вы понимаете, что дело не в предмете одежды.

Ответ : слой краски

23. Надо передать тебе

Загадка : У кого четыре пальца и большой палец, но он не жив?

Нет, ответ не «рука».

Ответ : Перчатка

24.Весна

Загадка : Что можно увидеть в середине марта и апреле, чего нельзя увидеть в начале или в конце каждого месяца?

Уловка здесь в том, что этот ответ не имеет ничего общего с временем года, когда выпадают месяцы, но с самими словами.

Ответ : Буква «Р»

25. Образцы насилия

Загадка : Женщина стреляет в своего мужа. Она погружает его под воду на несколько минут, а затем вешает.Сразу после этого они наслаждаются прекрасным ужином. Что случилось?

Когда мы говорим «он», мы на самом деле не имеем в виду человека.

Ответ : Она сфотографировала его и проявила в своей темной комнате.

26. Семейное время

Загадка : A – брат B. B – брат C. C – отец D. Итак, как D связан с A?

Пусть все эти разговоры о «братьях» и «отце» могут заставить вас задуматься о родственниках-мужчинах.

Ответ : A – тетя D.

27. Хорошая резка

Загадка : Мясник ростом шесть футов и один дюйм носит обувь 12-го размера. Что он весит?

Обратите особое внимание на формулировку вопроса – есть причина, по которой он не спрашивал: «Сколько он весит?»

Ответ : Мясо

28. Грязный вопрос

Загадка : Сколько грязи в яме размером четыре на четыре на пять футов?

Не позволяйте конкретным числам вводить вас в заблуждение – ответ будет одним и тем же независимо от того, какой размер отверстия измеряется.

Ответ : Нет

29. В поисках вдохновения

Загадка : Какое слово в английском языке соответствует всем следующим критериям: первые две буквы обозначают мужчину, первые три буквы обозначают женщину, первые четыре буквы обозначают великого человека, а все слово обозначает великую женщину .

Think Wonder Woman…

Ответ : Героиня

30. Некоторые находят это блаженным

Загадка : Что исчезает, когда вы произносите его имя?

Это также то, что вы можете “сломать” в момент, когда говорите.

Ответ : Тишина

СВЯЗАННЫЕ: Для получения более свежей информации подпишитесь на нашу ежедневный информационный бюллетень.

31. Я слышал, что

Загадка : Я говорю без уст и слышу без ушей. У меня нет тела, но я оживаю от ветра. Что я?

Не позволяйте описанию частей тела сбить вас с толку – ответ определенно не человек или животное. Чтобы найти ответ, сосредоточьтесь на «ветре».

Ответ : эхо

32.Один за другим

Загадка : Какие следующие три буквы в этой комбинации? ОТТФФСС.

Чтобы понять это, думайте о буквах меньше, чем о цифрах.

Ответ : ЛОР. Каждая буква – это первая буква числа, начинающегося с единицы. Итак, что будет после шести и семи? Восемь, девять и десять.

33. Обработка чисел

Загадка : Чем семерка отличается от остальных чисел от единицы до десяти?

В некотором роде противоположность предыдущей загадки, эта обманывает вас, заставляя думать, что это вопрос чисел, когда на самом деле речь идет о словах.

Ответ : Семь – единственный, состоящий из двух слогов.

34. Время на моей стороне

Загадка : Что происходит раз в минуту, два раза в мгновение, но никогда за тысячу лет?

Меньше думайте о значениях слов, чем о самих словах.

Ответ : Буква «м»

35. Psst… не угадаешь

Загадка : Если у тебя есть я, ты хочешь поделиться со мной. Если вы разделяете меня, у вас нет меня.Что я?

Это то, чем вы делитесь с друзьями.

Ответ : Секрет

36. Подписано, опечатано, доставлено

Загадка : У меня всего два слова, но тысячи букв. Что я?

«Письма» – важная подсказка, на которой здесь стоит сосредоточиться.

Ответ : Почтовое отделение

37. Съесть или быть съеденным

Загадка : Сначала съешь меня, потом тебя съедят. Что я?

«Вы» в данном случае – животное.

Ответ : рыболовный крючок

38. Многочисленные возможности

Загадка : Переверни меня на бок, и я все. Разрежь меня пополам, и я ничто. Что я?

Рассмотрим понятие формы.

Ответ : Число восемь

39. Путешественник

Загадка : Что может путешествовать по миру, оставаясь застрявшим в углу?

Также может оказаться в коллекции.

Ответ : штамп

40. Без головы

Загадка : У меня есть шея и нет головы, две руки и нет рук? Что я?

Давай, ты уже знаешь это.

Ответ : рубашка

9 головоломок и загадок, чтобы бросить вызов своим друзьям и семье

Поскольку головоломки часто требуют, чтобы вы напрягали свой мозг нестандартными способами и мыслили нестандартно, они являются отличным способом проверить свою остроту ума, сохранить сильную память, снизить стресс с помощью осознанной игры и, конечно же, получить массу удовольствия. по пути.

Вот 9 головоломок, логических задач и загадок с ответами, которые помогут вашим друзьям и близким. Этот сборник хитрых загадок включает в себя множество головоломок разной степени сложности. Есть легкие загадки для детей и более сложные головоломки для взрослых.

Мы включили решения к каждой загадке. Но сначала попробуйте бросить вызов самому себе. Сколько из этих головоломок и загадок вы сможете разгадать без читерства?

1. Загадка о переходе через реку (проблема с волком, козой и капустой)

Фермер идет домой с волком, козой и капустой, которые он купил на местном рынке.Он приходит к переправе через реку, и ему нужно арендовать лодку, чтобы переправиться через реку. Проблема в том, что эта лодка достаточно велика, чтобы вместить себя и разовую покупку. Если оставить без присмотра: коза съест капусту или волк съест козу. Фермер должен находиться в лодке, чтобы вести его.

Итак, как фермеру безопасно переправить все три предмета и самого себя через реку?

ОТКРОЙТЕ ОТВЕТ

Эта знаменитая загадка существует уже много веков и требует как творческого мышления, так и планирования задач.Оба этих когнитивных навыка позволяют осознать, что фермер может вернуть вещи на обратном пути после того, как пересечет реку. Хотя это (намеренно) не указано в истории, это также не запрещено прямо. Как только это станет известно, загадку станет легко разгадывать как дети, так и взрослые.

Фермер сначала ведет козу, а затем возвращается за капустой. Как только капуста будет перенесена на другую сторону, он заберет козу с собой, чтобы она не съела капусту.Он оставит козу на исходной стороне реки, а волка переправит. Поскольку волка можно безопасно оставить с капустой, он может легко вернуться еще раз, чтобы схватить козу, перенести ее на другую сторону и вернуться домой со всеми тремя целыми.


2. Таинственный круиз

Идя по реке, вы приближаетесь к лодке. Эта лодка не затонула. Но если вы присмотритесь, то увидите, что на лодке нет ни одного человека. Как это возможно?

ОБНАРУЖИТЬ ОТВЕТ

Все на лодке женаты.

Как и многие сложные загадки, эта головоломка была разработана, чтобы побудить логическую часть вашего мозга прыгнуть вперед. Наш мозг не любит пробелы в информации, поэтому ему нравится находить наиболее логичный ответ или решение, прежде чем изучать все данные. Вот почему многие люди почти сразу скажут «они спустились под палубу» или «прыгнули за борт».

Однако, как и в большинстве головоломок со словами, вы должны обращать внимание на каждую деталь. Сосредоточившись на слове «одиночный» и проанализировав другие его значения, вы сможете разгадать код этой сложной загадки.


3. Гонки для Королевства

Понимая, что его правление близится к концу, стареющий король изо всех сил пытался решить, кому из двух своих сыновей-близнецов также отдать свое королевство. Поэтому он предложил им посоревноваться в скачках, чтобы решить, кто будет следующим королем – с довольно своеобразной оговоркой: «Тот, кто последним пересечет финишную черту, получит королевство». _

Как и следовало ожидать, оба сына очень медленно начали гонку, пока наконец не наткнулись на мужчину на обочине дороги.Когда мужчина медленно шел рядом с ними, они объяснили, что делают. Затем он наклонился и дал два совета. После этого оба сына как можно быстрее устремились к финишу. Что это были за два совета?

ПОКАЗАТЬ ОТВЕТ

Переключить лошадей. Король указал, что ЛОШАДЬ, который последним пересечет финишную черту, будет победителем. Таким образом, меняя лошадей, каждый сын гарантировал, что победа в скачке на ДРУГОЙ лошади будет означать, что ИХ лошадь фактически пересечет финишную черту последней.

Эта головоломка поразила вас? Благодаря использованию формата рассказа с множеством не относящихся к делу деталей, он был разработан, чтобы заставить ваш мозг упускать из виду одну ключевую деталь, которая позволяет легко разгадать эту загадку.


4. Трое детей

У матери Джонни было трое детей. Первого ребенка назвали «Апрель». Второго ребенка назвали «Май». Как звали третьего ребенка?

ОТВЕТИТЬ НА ОТВЕТ

Johnny.

Честно говоря, хотя решение этой загадки очевидно, если вы внимательно ее прочитаете, вы изначально думали, что ответ был «июнь»? Это потому, что наш мозг любит упрощать сложную работу, которую он постоянно выполняет, выявляя закономерности и связи.Хотя в большинстве сценариев это действительно потрясающе, но, как видите, иногда это может нас подвести.


5. Проблема рая или ада

Вы стоите перед двумя большими дверями, охраняемыми большими солдатами. Одна дверь ведет вас на путь в рай, а другая ведет к вечному проклятию в аду. Вы можете задать один охранник, один вопрос (и только один вопрос). Вы знаете, что один солдат всегда говорит правду, а другой всегда лжет, но вы не уверены, кто из них какой.

ОТКРОЙТЕ ОТВЕТ

«Если я спрошу другого стражника, с какой стороны ведет на небеса, что они ответят?»

Прославленная в блокбастере « Лабиринт », эта головоломная логическая головоломка веками использовалась, чтобы бросить вызов друзьям и семье. Он использует хитрый вопрос, чтобы ответить на хитрый вопрос, что делает его довольно популярным среди любителей головоломок.

Предположим, левая дверь ведет на небеса.

Если бы вы прямо спросили «правдивого», какая дверь ведет на небеса, он бы ответил «налево», и, наоборот, лжец ответил бы «правильно».Поскольку вы не знаете, кто говорит правду, а кто лжет, дополнительная оговорка в вашем вопросе становится важной.

Если вы спросите «правду», он скажет вам правду о том, что сказал бы «лжец». В данном случае «правильно». И наоборот, если вы спросите «лжеца», он солгает о том, что сказал бы «рассказчик правды». Таким образом, вместо того, чтобы сказать «левый», «лжец» также сказал бы «право» в этой ситуации. Итак, кого бы вы ни спросили, вы всегда будете знать, какая дверь ведет в ад (или любую другую опасную ситуацию), и сможете пройти через противоположную дверь в безопасное место.


6. Унесенные ветром

Вы измеряете мою жизнь часами, а я служу вам истечением срока. Я быстр, когда худой, и медленнее, когда толстый. Ветер – мой враг.

ПОКАЗАТЬ ОТВЕТ

Свеча.

Эта забавная загадка требует, чтобы вы напрягли свой мозг и по-настоящему проанализировали, как каждая деталь работает вместе, чтобы создать неожиданное решение. Самый большой намек – это последнее предложение о ветре.


7.Загадка выключателя света (один свет и три выключателя)

Вы стоите перед закрытой комнатой с лампочкой внутри. Дверь закрыта. Он спускается к полу и не имеет окон, поэтому вы не можете заглянуть внутрь или узнать, включен или выключен свет. Однако вам говорят, что свет не горит, чтобы начать.

Перед вами три выключателя света, один из которых управляет светом. Вам нужно определить, какой именно. Вы можете переключать переключатели, как хотите.Но открыть дверь можно только один раз. И, как только вы это сделаете, вы больше не сможете прикасаться к переключателям. Как понять, какой переключатель управляет лампочкой?

НАЙТИ ОТВЕТ

Это популярная загадка для детей и взрослых. Хотя в нем не используется сложная логическая теория или математика, он требует от вас расширения вашего творческого мышления. Вы должны щелкнуть выключателем №1 и оставить его включенным на несколько минут. Затем переключите его обратно в исходную точку, быстро включите # 2, а затем сразу же откройте дверь и войдите.

Если индикатор горит, значит, им управляет переключатель №2. Если свет не горит, иди пощупай. Если лампочка горячая или теплая, значит, ею управляет переключатель №1. Однако, если свет не горит и холодный на ощупь, то единственный другой вариант – это управление переключателем №3.


8. Дневная перегородка

Алексис родилась 31 декабря, но всегда отмечает свой день рождения летом. Почему?

ПОКАЗАТЬ ОТВЕТ

Она живет в Южном полушарии.

Если вы живете в Австралии, Бразилии или где-нибудь еще к югу от экватора, вы, вероятно, очень запутались, почему это вообще загадка. Однако это может быть довольно сложно для тех, кто живет в Северном полушарии, где декабрь всегда ассоциируется с зимой. Тот факт, что канун Нового года был выбран в качестве ее дня рождения, также заставляет наш мозг задумываться и пытаться создать неясную связь или шаблон, которого не существует.

Ответить на эту загадку так легко, что на самом деле это сложно.Это похоже на то, как вы отвечаете «C» несколько раз подряд в тесте, и, несмотря на то, что вы знаете, что ответ на вопрос – «C», вы слишком задумываетесь и меняете ответ.


9. Возраст – это просто число

В 1990 году человеку исполняется 15 лет. В 1995 году этому человеку исполняется 10 лет. Как это возможно?

ОТКРОЙТЕ ОТВЕТ

С этим мы идем в доисторическую эпоху! На самом деле эта головоломка проще, чем вы думаете.Этот человек родился в 2005 году до нашей эры.

Итак, в отличие от лет н.э., которые начинаются с 0 года до н.э. лет назад, к году 0. Итак, 1995 г. до н. э. фактически на 5 лет раньше, чем в 1990 г. до н. э.

Итак, сколько из этих головоломок вы выполнили правильно?

В конце концов, это действительно не имеет значения. Эти загадки не предназначены для проверки интеллекта, чтобы вы выглядели гением или чтецом мыслей. Напротив, подобные головоломки и загадки – забавный способ вывести ваш мозг за рамки обычного мышления.Это лишь одна из многих причин, по которым вам следует тренировать свой мозг.

Пришло время поделиться этими головоломками и загадками со своими друзьями и семьей и проверить их!

Просмотреть все пазлы

Полезные ссылки по нашим волшебным изделиям

.

Добавить комментарий

Ваш адрес email не будет опубликован. Обязательные поля помечены *